Deel XVI Getaltheorie (Hoofdstukken 1 en 2) (recto-verso)

56
Wiskunde In zicht een cursus wiskunde voor studierichtingen met component wiskunde derde graad algemeen secundair onderwijs geschreven door Koen De Naeghel Deel XVI Getaltheorie (Hoofdstukken 1 en 2) 17/01/2016

description

Onderdeel van Wiskunde in zicht, een cursus wiskunde voor studierichtingen met component wiskunde derde graad algemeen secundair onderwijs geschreven door Koen De Naeghel

Transcript of Deel XVI Getaltheorie (Hoofdstukken 1 en 2) (recto-verso)

Page 1: Deel XVI Getaltheorie (Hoofdstukken 1 en 2) (recto-verso)

Wiskunde In zicht

een cursus wiskunde voor

studierichtingen met component wiskundederde graad algemeen secundair onderwijs

geschreven door

Koen De Naeghel

Deel XVI Getaltheorie (Hoofdstukken 1 en 2)

17/01/2016

Page 2: Deel XVI Getaltheorie (Hoofdstukken 1 en 2) (recto-verso)
Page 3: Deel XVI Getaltheorie (Hoofdstukken 1 en 2) (recto-verso)

Wiskunde In zicht

een cursus wiskunde voor

studierichtingen met component wiskundederde graad algemeen secundair onderwijs

geschreven door

Koen De Naeghel

Deel XVI Getaltheorie (Hoofdstukken 1 en 2)

Page 4: Deel XVI Getaltheorie (Hoofdstukken 1 en 2) (recto-verso)

CREATIVE COMMONS

Naamsvermelding-NietCommercieel-GelijkDelen 3.0(CC BY-NC-SA)

Dit is de vereenvoudigde (human-readable) versie van de volledige licentie.De volledige licentie is beschikbaar op de webpagina

http://creativecommons.org/licenses/by-nc-sa/3.0/nl/legalcode

De gebruiker mag:

het werk kopieren, verspreiden en doorgevenRemixen - afgeleide werken maken

Onder de volgende voorwaarden:

Naamsvermelding - De gebruiker dient bij het werk de door de maker of de licentiegever aangegeven naam tevermelden (maar niet zodanig dat de indruk gewekt wordt dat zij daarmee instemmen met je werk of je gebruik vanhet werk).Niet-commercieel - De gebruiker mag het werk niet voor commerciele doeleinden gebruiken.Gelijk delen - Indien de gebruiker het werk bewerkt kan het daaruit ontstane werk uitsluitend krachtens dezelfdelicentie als de onderhavige licentie of een gelijksoortige licentie worden verspreid.

Met inachtneming van:

Afstandname van rechten - De gebruiker mag afstand doen van een of meerdere van deze voorwaarden metvoorafgaande toestemming van de rechthebbende.Publiek domein - Indien het werk of een van de elementen in het werk zich in het publieke domein onder toepasselijkewetgeving bevinden, dan is die status op geen enkele wijze beınvloed door de licentie.Overige rechten - Onder geen beding worden volgende rechten door de licentie-overeenkomst in het gedrang gebracht:

• Het voorgaande laat de wettelijke beperkingen op de intellectuele eigendomsrechten onverlet.

• De morele rechten van de auteur.

• De rechten van anderen, ofwel op het werk zelf ofwel op de wijze waarop het werk wordt gebruikt, zoals hetportretrecht of het recht op privacy.

Let op - Bij hergebruik of verspreiding dient de gebruiker de licentievoorwaarden van dit werk kenbaar te maken aanderden. De beste manier om dit te doen is door middel van een link naar de webpaginahttp://creativecommons.org/licenses/by-nc-sa/3.0/nl/ .

Gepubliceerd door: Online uitgever Lulu.com

Auteursrecht omslagfoto: stylephotographs/123RF Stockfoto http://nl.123rf.com/profile stylephotographs

Tekstzetsysteem: LATEX

Royalty percentage: 0%

c© 2013 Koen De Naeghel

Gelicenseerd onder een Creative Commons Naamsvermelding-NietCommercieel-GelijkDelen 3.0

Druk 17 januari 2016

Page 5: Deel XVI Getaltheorie (Hoofdstukken 1 en 2) (recto-verso)

Inhoudsopgave Wiskunde In zicht

Voorwoord v

Wat is wiskunde? vii-xii

Parate kennis bij aanvang van de derde graad xiii-xxi

I Precalculus 1 I,i-ii,1-138

II Goniometrie en precalculus 2 II,i-ii,1-95

III Matrices III,i,1-81

IV Complexe getallen

V Logica

VI Rijen

VII Limieten, asymptoten en continuıteit

VIII Afgeleiden

IX Telproblemen

X Kansrekenen 1

XI Integralen

XII Ruimtemeetkunde

XIII Beschrijvende statistiek

XIV Kansrekenen 2 en verklarende statistiek XIV,i,1-39

XV Vectorvlak en Euclidisch vlak

XVI Getaltheorie (Hoofdstukken 1 en 2) XVI,i,1-33

XVII Vectorruimten en lineaire afbeeldingen

XVIII Analytische meetkunde

XIX Differentiaalvergelijkingen

XX Reeksen

G Computermeetkundepakket GeoGebra

M Computerrekenpakket Maple S,1-15

Po Portfolio wiskunde Po,1-4

Pr Practicum wiskunde

Ps Problem Solving wiskunde Pr,1-12

+∞ Topics uit de wiskunde +∞,1-5

Referentielijst, bibliografie en websites xxii-xxix

iii

Page 6: Deel XVI Getaltheorie (Hoofdstukken 1 en 2) (recto-verso)
Page 7: Deel XVI Getaltheorie (Hoofdstukken 1 en 2) (recto-verso)

Deel XVI

Algebra - Getaltheorie

XVI

Page 8: Deel XVI Getaltheorie (Hoofdstukken 1 en 2) (recto-verso)
Page 9: Deel XVI Getaltheorie (Hoofdstukken 1 en 2) (recto-verso)

Inhoudsopgave Deel Getaltheorie

1 Inleidende begrippen en definities 1

1.1 Verzamelingen . . . . . . . . . . . . . . . . . . . . . . . . . . . . . . . . . . . . . . . . . . . . . . . . . . . . . . . 1

1.2 Eigenschappen van natuurlijke en gehele getallen . . . . . . . . . . . . . . . . . . . . . . . . . . . . . . . . . . . . 3

Optelling van getallen - De groep Z,+ . . . . . . . . . . . . . . . . . . . . . . . . . . . . . . . . . . . . . . . . . . 3

Optelling van getallen - De ring Z,+, · . . . . . . . . . . . . . . . . . . . . . . . . . . . . . . . . . . . . . . . . . . 3

Ordening van getallen - De geordende ring Z,+, ·,≤ . . . . . . . . . . . . . . . . . . . . . . . . . . . . . . . . . . 4

Principe van inductie . . . . . . . . . . . . . . . . . . . . . . . . . . . . . . . . . . . . . . . . . . . . . . . . . . . 5

Principe van welordening - De welgeordende verzameling N,≤ . . . . . . . . . . . . . . . . . . . . . . . . . . . . 5

Oefeningen . . . . . . . . . . . . . . . . . . . . . . . . . . . . . . . . . . . . . . . . . . . . . . . . . . . . . . . . . . . . 6

2 Deelbaarheid van gehele getallen 8

2.1 Deelbaarheid en delers . . . . . . . . . . . . . . . . . . . . . . . . . . . . . . . . . . . . . . . . . . . . . . . . . . . 8

2.2 Deling met rest . . . . . . . . . . . . . . . . . . . . . . . . . . . . . . . . . . . . . . . . . . . . . . . . . . . . . . . 10

2.3 Veelvouden en lineaire combinaties . . . . . . . . . . . . . . . . . . . . . . . . . . . . . . . . . . . . . . . . . . . . 11

2.4 Eenheden, priemgetallen en samengestelde getallen . . . . . . . . . . . . . . . . . . . . . . . . . . . . . . . . . . . 12

2.5 Hoofdstelling van de rekenkunde . . . . . . . . . . . . . . . . . . . . . . . . . . . . . . . . . . . . . . . . . . . . . 14

2.6 Grootste gemene delers . . . . . . . . . . . . . . . . . . . . . . . . . . . . . . . . . . . . . . . . . . . . . . . . . . 16

2.7 Kleinste gemene veelvouden . . . . . . . . . . . . . . . . . . . . . . . . . . . . . . . . . . . . . . . . . . . . . . . 18

2.8 Toepassingen . . . . . . . . . . . . . . . . . . . . . . . . . . . . . . . . . . . . . . . . . . . . . . . . . . . . . . . . 19

Toepassing 1 - Fermatgetallen . . . . . . . . . . . . . . . . . . . . . . . . . . . . . . . . . . . . . . . . . . . . . . 19

Toepassing 2 - Mersennegetallen . . . . . . . . . . . . . . . . . . . . . . . . . . . . . . . . . . . . . . . . . . . . . 21

Toepassing 3 - Tandwielen . . . . . . . . . . . . . . . . . . . . . . . . . . . . . . . . . . . . . . . . . . . . . . . . 22

Oefeningen . . . . . . . . . . . . . . . . . . . . . . . . . . . . . . . . . . . . . . . . . . . . . . . . . . . . . . . . . . . . 23

Antwoorden op geselecteerde oefeningen 31

Page 10: Deel XVI Getaltheorie (Hoofdstukken 1 en 2) (recto-verso)
Page 11: Deel XVI Getaltheorie (Hoofdstukken 1 en 2) (recto-verso)

Hoofdstuk 1

Inleidende begrippen en definities

In dit eerste hoofdstuk1 definieren we enkele belangrijke begrippen en leggen we de notatie en terminologie vast diein het vervolg van dit deel voortdurend zal gebruikt worden. De definities en eigenschappen zijn erg eenvoudig, zodatdit hoofdstuk grotendeels als zelfstudie aan de leerling kan worden overgelaten.

1.1 Verzamelingen

In deze paragraaf overlopen we de belangrijkste kenmerken van een van de meest fundamentele begrippen in dewiskunde: verzamelingen.

3 Afspraak. Een verzameling kan omschreven worden als een collectie objecten, die we haar elementen noe-men. Een verzameling heeft geen ordening en elk element kan hoogstens een keer voorkomen. We kunnen eenverzameling beschrijven door haar elementen in een willekeurige volgorde op te sommen. Het geheel van alleelementen wordt tussen accolades geplaatst. We stellen een verzameling schematisch voor aan de hand van eenVenndiagram2.

Voorbeeld. A = {−2, 1, 3} is een verzameling. Omdat een verzameling geen ordening heeft, zal bijvoorbeeld{3,−2, 1} dezelfde verzameling A zijn. De verzameling A kan als volgt worden voorgesteld:

A

−213

3 Notatie. Beschouw een verzameling A. Als een element a tot A behoort, dan schrijven we a ∈ A, wat we lezenals a is een element van A. In het andere geval schrijven we a /∈ A. Bevat A een eindig aantal elementen, dannoteren we dat aantal met #A.

3 Definitie. De unieke verzameling zonder elementen noemen we de lege verzameling, genoteerd met {} of met ∅.3 Definitie. De verzameling van de natuurlijke getallen is {0, 1, 2, 3, . . .} en wordt genoteerd met N. We maken

de afspraak dat 0 een element is van N en we noteren de verzameling {1, 2, 3, . . .} als N0. De verzameling van degehele getallen is {0, 1,−1, 2,−2, . . .}, genoteerd met Z.

In symbolen:

N = {0, 1, 2, 3, . . .}N0 = {1, 2, 3, . . .}Z = {0, 1,−1, 2,−2, 3,−3, . . .}

3 Afspraak. Een andere mogelijkheid om een verzameling te beschrijven is het geven van een of meerdereeigenschappen waaraan de elementen van die verzameling moeten voldoen. De verticale streep | wordt gelezenals waarvoor geldt.

Voorbeeld. De verzameling {a ∈ Z | a2 = 4} is dezelfde als {2,−2}.3 Definitie. Zij A en B verzamelingen. Dan zeggen we dat A een deelverzameling is van B, genoteerd met A ⊆ B,

indien elk element van A ook een element is van B.

In symbolen:

A ⊆ B ⇔ ∀x ∈ A : x ∈ BIn dat geval zeggen we: de verzameling B omvat de verzameling A. Is dat niet zo, dan schrijven we A 6⊆ B.

1Dit hoofdstuk is gebaseerd op [68] (met toestemming van beide auteurs) en andere of eerdere uitgaven van Wiskunde In zicht.2Genoemd naar John Venn 1880. Gelijkaardige diagrammen werden eerder gebruikt door Leonhard Euler 18e eeuw, Gottfried

Willhelm Leibniz 17e eeuw en Ramon Llull 13e eeuw.

XVI-1

Page 12: Deel XVI Getaltheorie (Hoofdstukken 1 en 2) (recto-verso)

Voorbeeld. Elk natuurlijk getal is een geheel getal, dus de verzameling van alle natuurlijke getallen is eendeelverzameling van de verzameling van alle gehele getallen. Anders gezegd: De verzameling van de gehelegetallen omvat de verzameling van de natuurlijke getallen. In symbolen: N ⊆ Z.

Schematisch:

N012. . .

Z

−1−2. . .

3 Opmerking. Beschouw een verzameling A. Omdat de lege verzameling geen enkel element bevat, is voldaanaan de uitspraak ∀x ∈ ∅ : x ∈ A. Dus ∅ ⊆ A, met andere woorden: elke verzameling omvat de lege verzameling.

Hierboven hebben we al aangegeven dat {−2, 1, 3} en {3,−2, 1} dezelfde verzameling voorstellen. De volgende definitiegeeft betekenis aan het begrip gelijkheid van verzamelingen. Daarnaast kunnen we met de bewerkingen doorsnede,unie en verschil nieuwe verzamelingen maken.

3 Definitie. Twee verzamelingen A en B zijn gelijk als A een deelverzameling is van B en als B een deelverzamelingis van A. We schrijven dan A = B. Is dat niet zo, dan schrijven we A 6= B.

3 Definitie. Zij A en B verzamelingen. De doorsnede van A en B is de verzameling van alle elementen die in Aen in B bevat zijn. We noteren die verzameling met A ∩B.

In symbolen:

A ∩B = {x | x ∈ A en x ∈ B}

Schematisch:

A ∩B

A B

3 Definitie. De unie van A en B, genoteerd met A ∪ B, is de verzameling van alle elementen die in A of in Bbevat zijn.

In symbolen:

A ∪B = {x | x ∈ A of x ∈ B}

Schematisch:

A ∪B

A B

3 Definitie. Het verschil van A met B is de verzameling van alle elementen die wel tot A maar niet tot B behoren.Die verzameling wordt met A \B genoteerd.

In symbolen:

A\B = {x | x ∈ A en x /∈ B}

Schematisch:

A\B

A B

XVI-2

Page 13: Deel XVI Getaltheorie (Hoofdstukken 1 en 2) (recto-verso)

1.2 Eigenschappen van natuurlijke en gehele getallen

In deze paragraaf overlopen we de meest elementaire eigenschappen van de gehele getallen. De lezer kan zelf nagaanwelke van deze eigenschappen ook voor de natuurlijke getallen gelden.

Optelling van getallen - De groep Z,+

3 Eigenschap 1. De optelling van de gehele getallen voldoet aan de volgende eigenschappen:

(1) optelling is inwendig: ∀a, b ∈ Z : a+ b ∈ Z(2) optelling is associatief: ∀a, b, c ∈ Z : (a+ b) + c = a+ (b+ c)

(3) het getal 0 is het neutraal element voor optelling: ∀a ∈ Z : a+ 0 = a = 0 + a

(4) invers element voor optelling: ∀a ∈ Z : a+ (−a) = 0 = (−a) + a.

Omdat voldaan is aan eigenschappen (1)-(4) noemen we de verzameling Z voorzien van de optelling een groep,notatie Z,+. Bovendien geldt ook de eigenschap

(5) optelling is commutatief: ∀a, b ∈ Z : a+ b = b+ a.

Wegens eigenschap (5) noemen we de groep Z,+ commutatief (of abels3). Omdat Z,+ een groep is, geldt dezogenaamde

(6) schrapwet voor de optelling: ∀a, b, c ∈ Z : a+ b = a+ c⇒ b = c.

Bewijs van (6). Neem a, b, c ∈ Z. We hebben:

a+ b = a+ c ⇒ (−a) + (a+ b) = (−a) + (a+ c)

⇒ ((−a) + a) + b = ((−a) + a) + c

⇒ 0 + b = 0 + c

⇒ b = c.

Bij afspraak is het verschil a − b van twee gehele getallen a en b gelijk aan de som a + (−b). Hieruit volgen debekende tekenregels zoals a− (b+ c) = a− b− c en a− (b− c) = a− b+ c.

Vermenigvuldiging van getallen - De ring Z,+, ·3 Eigenschap 2. De vermenigvuldiging4 van de gehele getallen voldoet aan de volgende eigenschappen:

(7) vermenigvuldiging is inwendig: ∀a, b ∈ Z : a · b ∈ Z(8) vermenigvuldiging is associatief: ∀a, b, c ∈ Z : (a · b) · c = a · (b · c)(9) vermenigvuldiging is distributief ten opzichte van optelling: ∀a, b, c ∈ Z : a · (b+ c) = a · b+ a · c

∀a, b, c ∈ Z : (a+ b) · c = a · c+ b · c(10) vermenigvuldiging is commutatief: ∀a, b ∈ Z : a · b = b · a(11) het getal 1 is neutraal element voor vermenigvuldiging: ∀a ∈ Z : a · 1 = a = 1 · a.

Omdat de commutatieve groep Z,+ voldoet aan eigenschappen (7)-(11) noemen we de verzameling Z voorzienvan de optelling en de vermenigvuldiging een commutatieve ring met eenheidselement, notatie Z,+, ·. Bovendiengeldt ook de eigenschap

(12) er zijn geen nuldelers: ∀a, b ∈ Z : ab = 0⇒ a = 0 of b = 0.

Wegens eigenschap (12) noemen we de ring Z,+, · een integriteitsdomein, of kortweg domein. Omdat de ringZ,+, · een domein is, geldt ook de

(13) schrapwet voor de vermenigvuldiging: ∀a, b, c ∈ Z : ab = ac en a 6= 0⇒ b = c.

Bewijs van (13). Neem a, b, c ∈ Z. We hebben:

ab = ac ⇒ ab− ac = 0

⇒ a(b− c) = 0.

Vermits a 6= 0 moet dan b− c = 0 (waarom?) dus b = c.

3Genoemd naar Niels Henrik Abel (1802 - 1829).4De vermenigvuldiging van twee gehele getallen a en b wordt zowel met a · b als met ab genoteerd.

XVI-3

Page 14: Deel XVI Getaltheorie (Hoofdstukken 1 en 2) (recto-verso)

Ordening van getallen - De geordende ring Z,+, ·,≤3 Eigenschap 3. De relatie kleiner dan of gelijk aan van de gehele getallen voldoet aan de volgende eigenschappen:

(14) kleiner dan of gelijk aan is reflexief: ∀a ∈ Z : a ≤ a(15) kleiner dan of gelijk aan is antisymmetrisch: ∀a, b ∈ Z : a ≤ b en b ≤ a⇒ a = b

(16) kleiner dan of gelijk aan is transitief: ∀a, b, c ∈ Z : a ≤ b en b ≤ c⇒ a ≤ c.

Omdat voldaan is aan eigenschappen (14)-(16), noemen we de verzameling Z voorzien van kleiner of gelijk aaneen partieel geordende verzameling. Bovendien geldt ook de eigenschap

(17) kleiner dan of gelijk aan is totaal: ∀a, b ∈ Z : a ≤ b of b ≤ a.

Wegens eigenschap (17) noemen we de verzameling Z voorzien van kleiner of gelijk aan een totaal geordendeverzameling. Het ordenen van getallen is verenigbaar met de optelling en vermenigvuldiging, en wel op devolgende manier.

3 Eigenschap 4. Voor elke a, b, c ∈ Z geldt:

(18) als a ≤ b dan is a+ c ≤ b+ c

(19) als 0 ≤ a en 0 ≤ b dan is 0 ≤ ab.

Daarom spreken we van de geordende ring Z,+, ·,≤.

De relaties (strikt) kleiner dan, groter dan of gelijk aan en (strikt) groter dan kennen de volgende beschrijvingvoor alle gehele getallen a en b:

a < b ⇔ a ≤ b en a 6= b

a ≥ b ⇔ b ≤ aa > b ⇔ a ≥ b en a 6= b

De begrippen positief, negatief en hun negaties hebben de volgende betekenis voor alle gehele getallen a:

a is positief ⇔ a > 0

a is niet-positief ⇔ a ≤ 0

a is negatief ⇔ a < 0

a is niet-negatief ⇔ a ≥ 0

Voorbeeld. De verzameling van de positieve gehele getallen gelijk is aan N0, want

{a ∈ Z | a > 0} = {1, 2, 3, . . .} = N0.

Voor een geheel getal a wordt de absolute waarde van van a, genoteerd met |a|, als volgt gedefinieerd:

|a| ={

a als a ≥ 0−a als a < 0

Ten slotte vermelden we nog enkele andere eigenschappen van de geordende ring Z,+, ·,≤.

3 Gevolgen. Zij a, b, c ∈ Z. Dan geldt:

(20) als a ≤ b en 0 ≤ c dan is ac ≤ bc(21) als a ≤ b en c ≤ 0 dan is ac ≥ bc(22) |ab| = |a| · |b|(23) als a 6= 0 dan is |a| ≥ 1.

XVI-4

Page 15: Deel XVI Getaltheorie (Hoofdstukken 1 en 2) (recto-verso)

De partieel geordende verzameling N,≤ voldoet aan twee belangrijke eigenschappen die we in het vervolg regelmatigzullen gebruiken. Formeel gezien zijn deze principes equivalent, in die zin dat de ene uit de andere kan wordenaangetoond.

Principe van inductie

3 Eigenschap (principe van inductie). Zij V ⊆ N die voldoet aan de volgende twee voorwaarden:

(1) 0 ∈ V , en

(2) voor elke k ∈ N: als k ∈ V dan is k + 1 ∈ V .

Dan is V = N.

Deze eigenschap leidt tot een manier om eigenschappen van natuurlijke getallen te bewijzen.

3 Bewijs met inductie. Een eigenschap die geldt voor elk natuurlijk getal n ≥ n0 wordt vaak elegant aangetoonddoor middel van een bewijs met inductie op n. Zo’n bewijs bestaat uit twee delen. In het eerste deel, dat deinductiebasis wordt genoemd, wordt aangetoond dat de eigenschap geldt voor n = n0. In het tweede deel, deinductiestap, wordt aangetoond dat indien de eigenschap geldt voor een bepaalde waarde van n (bijvoorbeeldn = k) hij ook geldt voor de volgende waarde van n (dit is dan n = k + 1). (De veronderstelling dat de tebewijzen eigenschap geldt voor n = k noemen we de inductiehypothese.)

Als we dit hebben kunnen aantonen, dan is de stelling bewezen. Immers, we kunnen als volgt redeneren. Deeigenschap is waar voor n0 (dat was de inductiebasis) dus volgt uit de inductiestap dat de eigenschap waar isvoor n0 + 1. Maar dan volgt, opnieuw wegens de inductiestap, dat de eigenschap waar is voor n0 + 2, enzovoort.Daaruit volgt dat de eigenschap waar is voor alle n ≥ n0.

3 Modelvoorbeeld. Bewijs dat voor alle positieve natuurlijke getallen n geldt dat

1 + 2 + 4 + 8 + · · ·+ 2n−1 = 2n − 1.

Oplossing. We bewijzen de formule voor alle n ≥ 1 met inductie op n.

(i) Inductiebasis. Voor n = 1 is enerzijds 1 + 2 + 4 + 8 + · · ·+ 2n−1 = 1, terwijl anderzijds 2n − 1 = 2− 1 = 1.

(ii) Stel dat de formule waar is voor n = k (met k ≥ 1), dus dat 1 + 2 + 4 + 8 + · · ·+ 2k−1 = 2k− 1. We moetenaantonen dat de formule ook waar is voor n = k + 1, dus dat 1 + 2 + 4 + 8 + · · ·+ 2k+1−1 = 2k+1 − 1.

Welnu,

1 + 2 + 4 + 8 + · · ·+ 2k+1−1 = 1 + 2 + 4 + 8 + · · ·+ 2k

= 1 + 2 + 4 + 8 + · · ·+ 2k−1 + 2k

= 2k − 1 + 2k

= 2 · 2k − 1

= 2k+1 − 1.

Uit (i) en (ii) volgt nu dat de formule geldt voor alle n ≥ 1.

Principe van welordening - De welgeordende verzameling N,≤3 Eigenschap (principe van welordening). Elke niet-lege deelverzameling V ⊆ N bevat een kleinste element.

Het principe van welordening is equivalent met het principe van inductie, dus elk probleem dat kan worden opgelostmet inductie kan ook worden opgelost met welordening, en omgekeerd. Bij wijze van illustratie bewijzen we een andereelementaire eigenschap van de verzameling van de natuurlijke getallen met behulp van het principe van welordening.

3 Eigenschap (Archimedes). Als a, b ∈ N0 dan bestaat er een positief natuurlijk getal n waarvoor na ≥ b.

Bewijs. Stel, uit het ongerijmde, dat er geen enkel positief natuurlijk getal n bestaat waarvoor na ≥ b. Metandere woorden, stel dat na < b voor elk positief natuurlijk getal n. Beschouw nu de verzameling

V = {b− na | n ∈ N0}.Uit onze veronderstelling volgt dat V ⊆ N en merk op dat V niet leeg is (waarom?). Wegens het principe van dewelordening bevat V een kleinste element x. Omdat x ∈ V is x = b−ma voor een zekere m ∈ N0. Maar dan is

b− (m+ 1)a = b−ma− a < b−maeen element van V dat kleiner is dan x, een strijdigheid. Dus onze veronderstelling dat er geen enkel positiefnatuurlijk getal n bestaat waarvoor na ≥ b fout is. Met andere woorden, er is een positief natuurlijk getal nwaarvoor na ≥ b.

XVI-5

Page 16: Deel XVI Getaltheorie (Hoofdstukken 1 en 2) (recto-verso)
Page 17: Deel XVI Getaltheorie (Hoofdstukken 1 en 2) (recto-verso)

Oefeningen

1 Inleidende begrippen en definities Basis Verdieping Uitbreiding? ?? ? ?? ? ??

1.1 Verzamelingen 12

3 45

1.2 Eigenschappen van natuurlijke en gehele getallen 67

8 8 8 9 10

Oefeningen bij §1.1

B Oefening 1. Stel A = {1, 3, 5, 6} en B = {2, 3, 4, 6}. Beschrijf telkens de verzameling door middel van opsomming.

(a) A ∩B

(b) A ∪B

(c) A \B

(d) B \A

(e) A \A

(f) A \ (A ∩B)

B Oefening 2. Waar of vals? Beoordeel de volgende uitspraken. Verklaar telkens je antwoord.

(a) ∅ = {0}

(b) 6 ∈ {6}

(c) {2, 4, 6, 8, 10} ⊆ N

(d) ∅ = {∅}

(e) ∅ ∈ {∅}

(f) {1, 3, 6, 7, 8} ⊆ {1, 3, 6, 7, 9}

(g) ∅ ⊆ {3, 5, 7}

(h) {0} ∈ Z

B? Oefening 3. Beschrijf telkens de gegeven verzameling door opsomming.

(a) {y ∈ Z | 2y2 = 50}

(b) {x ∈ N | ∃m ∈ Z : x = 2m− 5}

(c) {a ∈ Z | 3a2 = −12}

V Oefening 4. Zij P en Q twee deelverzamelingen van Z waarvoor P ∪Q = P . Wat kan je uit elk van de onderstaandegegevens besluiten over P en Q?

(a) P ∩Q = ∅

(b) P ∪Q = ∅

(c) P ∩ (Z \Q) = ∅

(d) (Z \ P ) ∪Q = ∅

V Oefening 5 (Vlaamse Wiskunde Olympiade 1987 eerste ronde).Als V = {a, b, {c, d}} dan geldt

(A) c ∈ V

(B) {c, d} ⊆ V

(C) {a, b, c, d} ⊆ V

(D) {{c, d}} ⊆ V

(E) {c} ∈ V

XVI-6

Page 18: Deel XVI Getaltheorie (Hoofdstukken 1 en 2) (recto-verso)

Oefeningen bij §1.2

B Oefening 6. Beschouw de verzameling V = {a ∈ Z | −5 < a ≤ 3}.

(a) Beschijf de verzameling V door opsomming.

(b) Bevat V een eindig aantal elementen? Zo ja, geef #A.

B Oefening 7. Waar of vals? Beoordeel de volgende uitspraken. Indien waar, bewijs. Indien vals, geef een tegenvoor-beeld.

(a) De verzameling N voorzien van de optelling is een groep.

(b) De verzameling Z voorzien van de aftrekking is een groep.

(c) Elk geheel getal heeft een invers element voor de vermenigvuldiging.

(d) Elk niet-positief geheel getal is negatief.

(e) Elk positief geheel getal is niet-negatief.

(f) Als a, b ∈ Z dan is |a+ b| = |a|+ |b|.

(g) De verzameling Z voorzien van de relatie kleiner dan of gelijk aan voldoet aan het principe van welordening.

Oefening 8. Bewijs telkens met inductie de formule voor alle positieve natuurlijke getallen n.

B? (a) 1 + 2 + 3 + 4 + · · ·+ n =1

2n(n+ 1)

B? (b) 1 + 3 + 5 + 7 + · · ·+ (2n− 1) = n2

B?? (c) 1 + 3 + 9 + 27 + · · ·+ 3n−1 =1

2(3n − 1)

V (d) (1 + 2 + 3 + · · ·+ n)2 = 13 + 23 + 33 + · · ·+ n3

V? Oefening 9 (Nederlandse Wiskunde Olympiade 1986 finale).Bewijs dat voor alle positieve gehele getallen n geldt:

1

1 · 2 +1

3 · 4 + · · ·+ 1

(2n− 1) · (2n)=

1

n+ 1+

1

n+ 2+ · · ·+ 1

2n.

U?? Oefening 10 (veeltermidentiteiten). Een identiteit is een uitdrukking van de vorm � = 4, die waar is voor allewaarden van a, b, . . . of x, y, . . . die in � of 4 voorkomen en waarvoor de opgave zin heeft. Is zowel het linkerlid alshet rechterlid een veelterm, dan noemen we die identiteit een veeltermidentiteit. Zo zijn de volgende merkwaardigeproducten alle veeltermidentiteiten in twee variabelen a en b:

(a+ b)2 = a2 + 2ab+ b2

(a− b)2 = a2 − 2ab+ b2

(a+ b)3 = a3 + 3ab2 + 3a2b+ b3

(a− b)3 = a3 − 3ab2 + 3a2b− b3

a2 − b2 = (a− b)(a+ b)

a3 − b3 = (a− b)(a2 + ab+ b2)

a3 + b3 = (a+ b)(a2 − ab+ b2)

Vervangen we in de laatste drie identiteiten de variabele b door 1 en noemen we a nu x, dan verkrijgen we veeltermi-dentiteiten in een variabele x:

x2 − 1 = (x− 1)(x+ 1)

x3 − 1 = (x− 1)(x2 + x+ 1)

x3 + 1 = (x+ 1)(x2 − x+ 1)

Bewijs telkens met inductie de veeltermidentiteit.

(a) xn − 1 = (x− 1)(xn−1 + xn−2 + · · ·+ x2 + x+ 1) voor alle n ∈ N0

(b) x2n+1 + 1 = (x+ 1)(x2n − x2n−1 + x2n−2 − x2n−3 + · · ·+ x2 − x+ 1) voor alle n ∈ N

XVI-7

Page 19: Deel XVI Getaltheorie (Hoofdstukken 1 en 2) (recto-verso)

Hoofdstuk 2

Deelbaarheid van gehele getallen

In dit hoofdstuk zien we de meest elementaire begrippen en fundamentele eigenschappen van deelbaarheid van gehelegetallen. Hierbij nemen priemgetallen een centrale plaats in. Een van de doelstellingen van dit hoofdstuk is om dezogenaamde hoofdstelling van de rekenkunde aan te tonen: elk geheel getal kan geschreven worden als een product vanpriemgetallen, en dit op essentieel een manier. Daarnaast bespreken we enkele bijzondere getallen: grootste gemenedelers, kleinste gemene veelvouden, Fermatgetallen en Mersennegetallen. Tot slot passen we deelbaarheid van gehelegetallen toe op tandwielen.

2.1 Deelbaarheid en delers

In deze paragraaf bespreken we de definitie en enkele basiseigenschappen van deelbaarheid van gehele getallen.

3 Definitie. Zij a, b ∈ Z met b 6= 0. We noemen a (geheel) deelbaar door b als er een q ∈ Z bestaat waarvoora = qb. In dat geval noemen we b een (gehele) deler van a, hetgeen we noteren als b | a.

In symbolen:

b | a ⇔ ∃q ∈ Z : qb = a

Is b geen deler van a, dan schrijven we b 6 | a. Deelbaarheid door 0 blijft ongedefinieerd, zodat de schrijfwijzen0 | a en 0 6 | a geen betekenis hebben en dus zinloze uitspraken zijn.

Voorbeeld. We hebben dat 3 | 15, 8 | −8 en 6 6 | 21.

Bij de volgende eigenschap bewijzen we enkel (iii), (v) en (viii). De bewijzen van basiseigenschappen (i), (ii), (iv), (vi)en (vii) worden als Oefening 9 voor de lezer gehouden.

3 Eigenschap (basiseigenschappen van deelbaarheid). Zij a, b, c, k, l ∈ Z en d, e ∈ Z0. Dan geldt:

(i) 1 | a en −1 | a(ii) a 6= 0⇒ a | 0, a | a en −a | a(iii) d | a⇒ −d | a(iv) d | a⇒ d | ka en ed | ea(v) d | a en d | b⇒ d | (ka+ lb)

(vi) d | a en d | b⇒ d | ab(vii) d | e en e | a⇒ d | a(viii) d | a⇒ |d| ≤ |a| of a = 0.

Bewijs van (iii). Veronderstel dat d | a. We moeten aantonen dat −d | a. Daarvoor moeten we aantonen dat ereen geheel getal q bestaat waarvoor q(−d) = a.

Uit de veronderstelling dat d | a volgt dat er een geheel getal q′ bestaat waarvoor q′d = a. Dan is

a = q′d = (−q′)(−d)

zodat er inderdaad een geheel getal q bestaat waarvoor a = q(−d).

Bewijs van (v). Veronderstel dat d | a en d | b. We moeten aantonen dat d | (ka + lb). Daarvoor moeten weaantonen dat er een geheel getal q bestaat waarvoor qd = ka+ lb.

Uit de veronderstelling dat d | a volgt dat er een geheel getal q′ bestaat waarvoor q′d = a. Analoog is q′′d = bvoor een zeker geheel getal q′′. Dan is

ka+ lb = kq′d+ lq′′d = (kq′ + lq′′)d

zodat er inderdaad een geheel getal q bestaat waarvoor ka+ lb = qd.

XVI-8

Page 20: Deel XVI Getaltheorie (Hoofdstukken 1 en 2) (recto-verso)

Bewijs van (viii). Veronderstel dat d | a. We moeten aantonen dat |d| ≤ |a| of a = 0. Als a = 0 dan zijn weklaar. Veronderstel dus dat a 6= 0. We moeten aantonen dat |d| ≤ |a|.Uit d | a volgt dat a = qd voor een zekere q ∈ Z. Omdat a 6= 0 is ook q 6= 0, zodat |q| ≥ 1. Vermenigvuldigenwe beide leden van deze ongelijkheid met |d|, dan vinden we

|q| ≥ 1 ⇒ |q| · |d| ≥ |d|⇒ |qd| ≥ |d|⇒ |a| ≥ |d|

hetgeen te bewijzen was.

Is a ∈ Z0, dan volgt uit basiseigenschap (viii) van deelbaarheid dat elke deler d van a voldoet aan |d| ≤ |a|. Is |d| < |a|dan noemen we d een echte deler van a. Voor het vervolg is het handig om een notatie voor alle delers van een geheelgetal te voorzien.

3 Notatie. Zij a ∈ Z. De verzameling van alle delers van a wordt genoteerd met del(a).

In symbolen:

del(a) = {b | b ∈ Z0 en b | a}

Eens men de begrippen deelbaarheid en delers beheerst, dan kunnen typische gevolgen hiervan op eenvoudige manierbewezen worden.

3 Modelvoorbeeld.

(a) Bepaal del(1), del(−12), del(17) en del(0).

(b) We weten dat 6 | 12. Welk verband merk je op tussen de verzamelingen del(6) en del(12)?

(c) Formuleer en bewijs een veralgemening van (b) voor willekeurige gehele getallen.

Oplossing.

(a) We hebben:

del(1) = {1,−1},del(−12) = {1,−1, 2,−2, 3,−3, 4,−4, 6,−6, 12,−12},

del(17) = {1,−1, 17,−17},del(0) = Z0.

(b) del(6) ⊆ del(12)

(c) Zij a, b ∈ Z met b 6= 0. Dan geldt: b | a⇔ del(b) ⊆ del(a).

Bewijs. Stel dat b | a. Om aan te tonen dat del(b) ⊆ del(a), moeten we bewijzen dat elk element van del(b)bevat is in del(a). Zij c ∈ del(b). Dan is c ∈ Z0 en c | b. Uit basiseigenschap (vii) van deelbaarheid volgt nudat c | a, zodat c ∈ del(a). We besluiten dat del(b) ⊆ del(a).

Omgekeerd, stel dat del(b) ⊆ del(a). Omdat b ∈ del(b) geldt nu ook dat b ∈ del(a), zodat b | a.

Een beschrijving van het aantal delers van een geheel getal volgt uit de basiseigenschappen van deelbaarheid.

3 Gevolg. Zij a ∈ Z. Dan geldt:

(i) del(a) 6= ∅,(ii) als a = 0 dan is del(a) = Z0,

(iii) als a 6= 0 dan bevat del(a) een eindig en even aantal elementen.

Bewijs. (i) Uit basiseigenschap (i) van deelbaarheid volgt dat 1 ∈ del(a), zodat del(a) 6= ∅.(ii) Uit de definitie van deelbaarheid volgt dat elk geheel getal b 6= 0 een deler van 0 is, want 0 · b = 0.

(iii) Stel dat a 6= 0. Is b ∈ del(a), dan volgt uit basiseigenschap (viii) van deelbaarheid dat −a ≤ b ≤ a. Dittoont aan dat del(a) een deelverzameling van een eindige verzameling is:

del(a) ⊆ {1,−1, 2,−2, . . . , a,−a}

zodat del(a) zelf ook zelf een eindige verzameling is. Verder, is b ∈ del(a) dan is b 6= 0 (definitie deelbaarheid)en is −b ∈ del(a) (zie basiseigenschap (iii) van deelbaarheid). Dit toont aan dat het aantal elementen vandel(a) even is.

XVI-9

Page 21: Deel XVI Getaltheorie (Hoofdstukken 1 en 2) (recto-verso)

2.2 Deling met rest

Is een geheel getal niet deelbaar door een ander geheel getal verschillend van nul, dan moeten we ons tevreden stellenmet een zogenaamde deling met rest, ook wel de Euclidische deling genoemd. Omdat dit een fundamenteel resultaatis, plaatsen we de formulering ervan in een stelling. Het bewijs levert een algoritme1 om deling met rest uit te voeren.

3 Stelling (deling met rest, Euclidische deling). Zij a, b ∈ Z met b 6= 0. Dan bestaat er precies een geheelgetal q en precies een geheel getal r zodat

a = qb+ r en 0 ≤ r < |b| .

Het getal a wordt het deeltal genoemd, b de deler, q het quotient en r de rest van de deling van a door b.

Bewijs. Beschouw de verzameling S van alle gehele getallen van de vorm a − nb waarbij n een geheel getal is.In symbolen:

S = {a− nb | n ∈ Z}= {a, a− b, a+ b, a− 2b, a+ 2b, a− 3b, a+ 3b, . . .}.

De verzameling S bevat minstens een natuurlijk getal (ga na). Wegens het principe van welordening bevat Seen kleinste natuurlijk getal, dat we met r noteren. Omdat r ∈ S geldt dat r = a − qb voor een zeker geheelgetal q. We beweren nu dat r < |b|. Inderdaad, mocht r ≥ |b| dan zou het natuurlijk getal r′ = r − |b| tot Sbehoren (ga na) waarbij r′ < r (waarom?) en dit is strijdig met het feit dat r het kleinste natuurlijk getal in Sis. Dus r < |b|. Op die manier hebben we aangetoond dat er gehele getallen q en r bestaan waarvoor a = qb+ ren 0 ≤ r < |b|.Om aan te tonen dat q en r uniek zijn, nemen we aan dat ook a = q′′b + r′′ en 0 ≤ r′′ < |b| voor zekere gehelegetallen q′′, r′′. Uit a = qb+ r en a = q′′b+ r′′ volgt dat (ga na):

(q − q′′)b = r′′ − r ⇒ b | r′′ − r.

Mocht r′′ 6= r dan zou uit basiseigenschap (8) van deelbaarheid volgen dat |b| ≤ |r′′ − r|. Anderzijds volgt uitr′′ < |b| en −r < 0 dat r′′ − r < |b| en analoog is ook r − r′′ < |b| zodat |r′′ − r| < |b|, een strijdigheid. Webesluiten dat r = r′′. Uit a = qb+ r en a = q′′b+ r volgt eenvoudig dat q = q′′ (doe dit). Daarmee is aangetoonddat er precies een geheel getal q en precies een geheel getal r bestaat waarvoor a = qb+ r en 0 ≤ r < |b|.

Merk op dat a ∈ Z deelbaar is door b ∈ Z0 als en slechts als de rest bij deling van a door b gelijk is aan nul. In datgeval spreken we van een opgaande deling. In het algemeen geeft het bewijs van de vorige stelling een manier omdeling met rest van twee gehele getallen uit te voeren.

3 Modelvoorbeeld. Bepaal het quotient en de rest van de deling van −45 door 14.

Oplossing. We schrijven −45 als de som van een geheel aantal keer 14 met een restgetal, en manipuleren hetveelvoud van 14 zodat het restgetal niet-negatief en kleiner dan 14 wordt.

−45 = 0 · 14 + (−45)

= (−1) · 14 + (−31)

= (−2) · 14 + (−17)

= (−3) · 14 + (−3)

= (−4) · 14 + 11 en 0 ≤ 11 ≤ |14| .

Dus de deling van −45 door 14 heeft als quotient q = −4 en als rest r = 11.

Zijn deeltal en deler positieve gehele getallen, dan kan het quotient en de rest bij deling van a door b sneller gevondenworden met de klassieke staartdeling uit het lager onderwijs. Bij de aanvankelijke opzet ging het daarbij om de delingvan een (meestal groot) natuurlijk getal door een kleiner natuurlijk getal. Het principe kan echter ook voor anderedelingen gebruikt worden, bijvoorbeeld voor de deling van veeltermen.

1Een algoritme is een eindige reeks instructies die vanuit een gegeven begintoestand naar een beoogd doel leiden. Het doel van eenalgoritme kan van alles zijn met een duidelijk resultaat. Eenzelfde taak kan gewoonlijk met verschillende reeksen instructies wordenopgelost. Het verschil ligt dan meestal in de hoeveelheid tijd, ruimte of inspanning die het algoritme vergt, dit is de complexiteit van eenalgoritme.

XVI-10

Page 22: Deel XVI Getaltheorie (Hoofdstukken 1 en 2) (recto-verso)

2.3 Veelvouden en lineaire combinaties

De begrippen veelvoud en deler houden nauw met elkaar verband.

3 Definitie. Zij a, b ∈ Z. We noemen a een (geheel) veelvoud van b als er een q ∈ Z bestaat waarvoor a = bq.

De gehele veelvouden van 2 zijn precies de even getallen, de overige gehele getallen zijn oneven. Voor het vervolg ishet handig om een notatie voor de veelvouden van een willekeurig geheel getal a te voorzien.

3 Notatie. Zij a ∈ Z. De verzameling van alle veelvouden van a noteren we met aZ.

In symbolen:

aZ = {aq | q ∈ Z}

Voorbeeld. We hebben (vul aan):

5Z = {0, 5,−5, 10,−10, 15,−15, 20,−20, . . .},

(−5)Z = {0,−5, 5,−10, 10,−15, 15,−20, 20, . . .} = 5Z,

1Z = Z,

0Z = {0}.

De verzameling aZ van alle gehele veelvouden van een geheel getal a kan als volgt worden veralgemeend.

3 Definitie. Zij a, b ∈ Z. Een (gehele) lineaire combinatie van a en b is een geheel getal van de gedaante

am+ bn waarbij m,n ∈ Z.

De verzameling van alle lineaire combinaties van a en b noteren we met aZ + bZ.

In symbolen:

aZ + bZ = {am+ bn | m,n ∈ Z}

Voorbeeld. We hebben (vul aan):

−20Z + 12Z = {−20m+ 12n | m,n ∈ Z}

= {−20, 12,−20 + 12,−20− 12, 20 + 12, 20− 12,−20 + 24,−20− 24, 20 + 24, 20− 24, . . .}

= {−20, 12,−8,−32, 32, 8, 4,−48, 48,−4,−28,−52, 52, 28, . . .}.We vermoeden dat −20Z + 12Z = . . .Z (vul aan). Hoe kunnen we dit aantonen?

Enerzijds is 4 = −20 · 1 + 12 · (−2) zodat 4q = −20q+ 12(−2q) voor elk geheel getal q, zodat 4Z ⊆ −20Z+ 12Z.Anderzijds, stel dat y = −20m+ 12n ∈ −20Z + 12Z. Omdat 4 | −20 en 4 | 12 is 4 | y zodat y ∈ 4Z.

De veralgemening van dit voorbeeld is cruciaal in het bewijs van een belangrijke stelling die we later zullen zien.Daarom plaatsen we dit resultaat onder de naam lemma, ook wel een hulpstelling genoemd.

3 Lemma 1. Zij a, b ∈ Z. Dan is er een geheel getal d waarvoor aZ + bZ = dZ.

Bewijs. Als a = 0 en b = 0 dan volstaat de keuze d = 0 (ga na), zodat we voor het vervolg van het bewijsmogen veronderstellen dat a en b niet beide nul zijn. De verzameling aZ+ bZ bevat minstens een positief geheelgetal (ga na). Wegens het principe van welordening bevat aZ + bZ een kleinste positief geheel getal, dat wemet d noteren. Omdat d ∈ aZ + bZ geldt dat d = ak + bl voor zekere gehele getallen k, l. We beweren nu dataZ + bZ = dZ. Daartoe bewijzen we dat dZ ⊆ aZ + bZ en aZ + bZ ⊆ dZ.

Neem x ∈ dZ. Dan is x = dq voor een zekere q ∈ Z. Nu is

x = dq = (ak + bl)q = akq + blq ∈ aZ + bZ.

Hiermee is aangetoond dat dZ ⊆ aZ + bZ.

Om aan te tonen dat aZ + bZ ⊆ dZ, nemen we y ∈ aZ + bZ. Dan is y = am+ bn voor zekere m,n ∈ Z. Wegensde Euclidische deling bestaat er precies een geheel getal q en precies een geheel getal r zodat

y = qd+ r en 0 ≤ r < d.

We beweren dat r = 0. Inderdaad, mocht r 6= 0 dan zou

r = y − qd = am+ bn− q(ak + bl) = a(m− qk) + b(n− ql) ∈ aZ + bZ

en dit is strijdig met het feit dat d het kleinste positief geheel getal in aZ + bZ is. We besluiten dat r = 0 zodaty = qd ∈ dZ, waarmee bewezen is dat aZ + bZ ⊆ dZ.

Analoog kan het volgend resultaat aangetoond worden. Het bewijs laten we als Oefening 36 voor de lezer.

3 Lemma 2. Zij a, b ∈ Z. Dan is er een geheel getal v waarvoor aZ ∩ bZ = vZ.

XVI-11

Page 23: Deel XVI Getaltheorie (Hoofdstukken 1 en 2) (recto-verso)

2.4 Eenheden, priemgetallen en samengestelde getallen

We hebben gezien dat elk geheel getal a 6= 0 een even aantal delers heeft, meer bepaald:

# del(a) ∈ {2, 4, 6, 8, . . .}.Gehele getallen met twee delers, zogenaamde eenheden, zijn eenvoudig te bepalen. Gehele getallen met vier delersworden priemgetallen genoemd. We zullen zien dat alle andere gehele getallen verschillend van nul kunnen geschrevenworden als een product van eenheden en priemgetallen. Daarom zullen we gehele getallen verschillend van nul met zesof meer delers samengestelde getallen noemen.

3 Eigenschap2. Zij a ∈ Z. Dan zijn de volgende uitspraken equivalent:

(i) # del(a) = 2,

(ii) a = 1 of a = −1,

(iii) er bestaat een b ∈ Z zodat ab = 1.

Zo’n geheel getal a wordt een eenheid van Z genoemd. De eenheden van Z zijn dus precies de getallen 1 en −1,en we noteren Z∗ = {1,−1}.

Bewijs. We bewijzen de equivalenties (i)⇔(ii) en (ii)⇔(iii).

(i)⇔(ii) Stel dat # del(a) = 2. Dan is a 6= 0 (waarom?). Uit basiseigenschappen (i) en (ii) van deelbaarheidvolgt nu dat {1,−1, a,−a} ⊆ del(a), zodat onvermijdelijk a = 1 of a = −1. Omgekeerd, stel dat a = 1 of a = −1.Omdat del(1) = {−1, 1} en del(−1) = {−1, 1} is # del(a) = 2.

(ii)⇔(iii) Er geldt: ∃b ∈ Z : ab = 1 ⇔ a | 1⇔ a ∈ del(1)

⇔ a = 1 of a = −1.

Euclides van Alexandrie(derde eeuw voor Christus)

3 Stelling (eerste stelling van Euclides)3. Zij p ∈ Z0 en p 6∈ Z∗. Dan zijn devolgende uitspraken equivalent:

(i) # del(p) = 4,

(ii) ∀a, b ∈ Z : p = ab⇒ a ∈ Z∗ of b ∈ Z∗,(iii) ∀a, b ∈ Z : p | ab⇒ p | a of p | b.

Zo’n geheel getal p wordt een priemgetal van Z genoemd.

Bewijs. We bewijzen de equivalenties (i)⇔(ii) en (ii)⇔(iii).

(i)⇒(ii) Stel dat # del(p) = 4. Dan is p 6= 0 (waarom?). Uit basiseigenschappen(i) en (ii) van deelbaarheid volgt nu dat {1,−1, p,−p} = del(p). Neem nua, b ∈ Z willekeurig en veronderstel dat p = ab. We moeten aantonen data ∈ Z∗ of b ∈ Z∗. Uit p = ab volgt dat a | p, zodat a ∈ del(p) = {1,−1, p,−p}.Als a = 1 of a = −1 dan is a ∈ Z∗. Als a = p of a = −p dan volgt uit p = abdat b = 1 resp. b = −1 dus b ∈ Z∗.(ii)⇒(i) Uit basiseigenschappen (i) en (ii) van deelbaarheid volgt alvast dat {−1, 1,−p, p} ⊆ del(p). We zullenaantonen dat ook del(p) ⊆ {−1, 1,−p, p}. Omdat p 6∈ Z∗ zal dan gelden dat # del(p) = #{−1, 1,−p, p} = 4.

Zij a ∈ del(p). Dan is p = aq voor een zekere q ∈ Z. Uit de veronderstelling volgt dat a ∈ Z∗ of q ∈ Z∗.Als a ∈ Z∗ dan is a = 1 of a = −1. Als q ∈ Z∗ dan volgt uit p = aq dat a = p of a = −p. Zodoende isa ∈ {1,−1, p,−p}, dus del(p) ⊆ {1,−1, p,−p}.(ii)⇒(iii) Neem a, b ∈ Z willekeurig en veronderstel dat p | ab. We moeten aantonen dat p | a of p | b.Onderstel dat p 6 | a. Wegens een eerder lemma bestaat er een geheel getal d waarvoor aZ + pZ = dZ. Nu isp = a · 0 + p · 1 ∈ aZ+ pZ = dZ zodat p = du voor een u ∈ Z. Dus u is een eenheid of d is een eenheid. Mocht ueen eenheid zijn dan zou dZ = pZ en dus a ∈ pZ wat in tegenspraak is met onze veronderstelling. Dus d is eeneenheid, zodat er gehele getallen k, l bestaan zodat ak + pl = 1. Uit akb+ plb = b en p | ab volgt dan dat p | b.(iii)⇒(ii) Neem a, b ∈ Z willekeurig en veronderstel dat p = ab. We moeten aantonen dat a ∈ Z∗ of b ∈ Z∗. Uitp = ab volgt dat p | ab en dus geldt p | a of p | b. Onderstel dat p | a dan is a = pq voor een zekere q ∈ Z. Dusgeldt p = pqb. Omdat p 6= 0 mogen we beide leden delen door p, zodat qb = 1. Dit bewijst dat b een eenheid is.Analoog volgt uit p | b dat a een eenheid is.

2In de eerste plaats wordt elk geheel getal dat aan (iii) voldoet een eenheid van Z genoemd. Uit deze eigenschap volgt dan dat deeenheden van Z precies de gehele getallen 1 en −1 zijn.

3In feite wordt enkel de implicatie (i)⇒(iii) de eerste stelling van Euclides genoemd, naar Euclides, Elementen, Boek VII, Propositie30. In de eerste plaats wordt elk geheel getal dat aan (ii) voldoet een irreducibel getal genoemd en elk geheel getal p dat aan (iii) voldoeteen priemgetal genoemd. Uit deze stelling volgt dan dat de irreducibele getallen van Z precies de priemgetallen van Z zijn.

XVI-12

Page 24: Deel XVI Getaltheorie (Hoofdstukken 1 en 2) (recto-verso)

Eratosthenes van Cyrene(c.276 v.Chr. - c.195 v.Chr.)

De eerste positieve priemgetallen zijn gelijk aan

2, 3, 5, 7, 11, 13, 17, 19, 23, 29, 31, 37, 41, 43, 47, 53, . . .

en uiteraard zijn de tegengestelde getallen ook priemgetallen: −2,−3,−5 etc. Er iseen eenvoudige manier om een tabel van (kleine) positieve priemgetallen op te stellendoor middel van een procedure die bekend staat als de zeef van Eratosthenes.

Stap 1. Schrijf alle natuurlijke getallen van 2 tot een met een zelf te kiezenmaximum N op.

Stap 2. Stel p = 2, het eerste positieve priemgetal.

Stap 3. Verwijder alle veelvouden 2p, 3p, 4p, 5p, . . . uit de lijst.

Stap 4. Bepaal het eerste getal dat groter is dan p.

(i) Als dit getal niet voorkomt in de lijst, dan stopt de procedure.

(ii) Als dat getal wel voorkomt, dan is dat getal een priemgetal en noemen wedit nieuw getal p. Herhaal de procedure nu vanaf Stap 3.

Deze werkwijze doet ons vermoeden dat er oneindig veel priemgetallen bestaan. Om dat resultaat aan te tonen,moeten we ons wenden tot getallen met meer dan vier delers. Gemakshalve geven we die een naam.

3 Definitie. Zij a ∈ Z0. We noemen a een samengesteld getal als # del(a) > 4.

Voorbeeld. De getallen 15,−6 en 918 zijn samengesteld. De getallen −7, 1 en 0 zijn niet samengesteld.

3 Lemma. Elk samengesteld getal is deelbaar door een priemgetal.

Zo’n priemgetal wordt een priemdeler van dat samengesteld getal genoemd.

Bewijs. Zij a een samengesteld getal. Dan is # del(a) > 4 zodat del(a) natuurlijke getallen tussen 1 en a bevat.Noem p de kleinste van die natuurlijke getallen. We beweren dat p een priemgetal is. Inderdaad, mocht p geenpriemgetal zijn dan zou, wegens de eerste stelling van Euclides, er een natuurlijk getal q tussen 1 en p bestaandie p deelt. Nu volgt uit q | p en p | a dat q | a, maar dit is in strijd met onze keuze van p. We besluiten dat peen priemgetal is dat a deelt.

3 Stelling (tweede stelling van Euclides)4. Er zijn oneindig veel priemgetallen.

Bewijs van Euclides. Veronderstel, uit het ongerijmde, dat er een eindig aantal priemgetallen zijn. Noem2, 3, 5, . . . , p alle positieve priemgetallen en noem

n = 2 · 3 · 5 · 7 . . . p+ 1.

Omdat 2, 3, 5, . . . , p alle positieve priemgetallen zijn en n > p, is n geen priemgetal. Dus is n een samengesteldgetal, en uit het vorige lemma volgt dat n deelbaar is door een priemgetal. Maar n is niet deelbaar door degetallen 2, 3, 5, . . . , p (waarom?), een strijdigheid. We besluten dat er oneindig veel priemgetallen zijn.

We kunnen het voorgaande lemma ook gebruiken om aan te tonen dat elk samengesteld getal kan geschreven wordenals een product van niet-samengestelde getallen. Zo is bijvoorbeld 918 = 2 · 33 · 17. We bewijzen dit resultaat vooreen willekeurig samengesteld getal.

3 Stelling. Elk samengesteld getal is een product van priemgetallen.

Bewijs. Zij a een samengesteld getal. We mogen aannemen dat a > 1. Wegens het vorige lemma is a deelbaardoor een positief priemgetal p1 zodat er een geheel getal a1 is met

a = p1a1 waarbij 1 < a1 < a.

Als a1 een priemgetal is, dan zijn we klaar. Anders is, wegens het vorige lemma, a1 een samengesteld getal endus deelbaar door een priemgetal p2 zodat er een geheel getal a2 is met

a = p1p2a2 waarbij 1 < a2 < a1 < a.

Herhalen van deze procedure levert een dalende rij van natuurlijke a1, a2, a3, . . . die alle groter dan 1 zijn. Dusdeze procedure stopt na een eindig aantal stappen, zodat a een product van priemgetallen is.

4Euclides, Elementen, Boek IX, Propositie 20.

XVI-13

Page 25: Deel XVI Getaltheorie (Hoofdstukken 1 en 2) (recto-verso)

2.5 Hoofdstelling van de rekenkunde

Een samengesteld getal kan op meerdere manieren als een product van priemgetallen geschreven worden. Zo isbijvoorbeeld

−68 = (−2) · 2 · 17 = 22 · (−17) = 2 · (−17) · 2 = 17 · 2 · (−2).

Door een eenheid voorop te schrijven, kunnen we ervoor zorgen dat de priemgetallen die in zo’n product voorkomenalle positief zijn. Door deze priemgetallen te rangschikken van klein naar groot, vinden we slechts een ontbinding:

−68 = (−1) · 22 · 17.

Nu geldt dat er voor elk samengesteld getal slechts een zo’n ontbinding is.

Carl Friedrich Gauss(1777-1855)

3 Stelling (hoofdstelling van de rekenkunde)5 Zij a ∈ Z0. Dan kan a opprecies een manier geschreven worden als

a = e pa11 p

a22 . . . pak

k

waarbij e ∈ Z∗, k ∈ N, ai ∈ N0 en p1 < p2 < . . . < pk positieve priemgetallenzijn. Deze unieke schrijfwijze noemen we de standaardontbinding van a.

Bewijs. Het is duidelijk dat de stelling waar is als a een eenheid is of als a eenpriemgetal is (ga na). Dus voor het vervolg van het bewijs mogen we aannemendat a een samengesteld getal is.

Wegens de vorige stelling kan a geschreven worden als een product van priemge-tallen. Door eventuele mintekens voorop te plaatsen en factoren te herschikken,kunnen we a alvast op minstens een manier schrijven als

a = e pa11 p

a22 . . . pak

k (∗)waarbij e ∈ Z∗, k ∈ N, ai ∈ N0 en p1 < p2 < . . . < pk positieve priemgetallen zijn. Om aan te tonen dat erhoogstens een manier is, veronderstellen we dat er nog een tweede manier is, met andere woorden

a = e′ qb11 qb22 . . . qbll (∗∗)

waarbij e′ ∈ Z∗, l ∈ N, bj ∈ N0 en q1 < q2 < . . . < ql positieve priemgetallen zijn. We moeten aantonen date = e′, k = l, pi = qi en ai = bi voor elke i.

Uit (∗) en (∗∗) volgte pa1

1 pa22 . . . pak

k = e′ qb11 qb22 . . . qbll

en omdat e, e′ eenheden zijn en elke pi en elke qj positief is, geldt dat e = e′. Dus

pa11 p

a22 . . . pak

k = qb11 qb22 . . . qbll .

Nu is p1 een deler van het linkerlid, en dus ook een deler van het rechterlid. Uit de eerste stelling van Euclidesvolgt dat p1 | qj voor een zekere j. Omdat qj een (positief) priemgetal is, volgt uit de eerste stelling van Euclidesdat p1 = qj . Analoog voor p2, . . . , pk zodat elk priemgetal aan de linkerkant ook aan de rechterkant voorkomt.Een gelijkaardig argument toont aan dat elk priemgetal aan de rechterkant ook aan de linkerkant voorkomt.Omdat p1 < p2 < . . . < pk en q1 < q2 < . . . < ql is k = l en pi = qi voor elke i. De bovenstaande gelijkheidwordt dan:

pa11 p

a22 . . . pak

k = pb11 pb22 . . . pbkk .

Er rest ons nu nog om aan te tonen dat ai = bi voor elke i. Mocht bijvoorbeeld a1 > b1 dan zou, na deling vanbeide leden door pb1 , gelden dat

pa1−b11 pa2

2 . . . pak

k = pb22 . . . pbkk .

zodat de linkerkant deelbaar is door p1 en de rechterkant niet, een strijdigheid. Met een gelijkaardig argumentvinden we dat a1 < b1 onmogelijk is, zodat a1 = b1. Analoog is ook a2 = b2, . . . , ak = bk.

Hoewel er verschillende technieken zijn om een geheel getal te ontbinden in priemfactoren, is er tot op heden geenefficient algoritme bekend. Net die onwetendheid garandeerd de veiligheid van heel wat hedendaagse codeermecha-nismen, waarvan vele afgeleid zijn van de codeermethode RSA6. Natuurlijke getallen n die relatief klein zijn kunnenhandmatig ontbonden worden met behulp van een lijst van priemgetallen tot en met

√n. Inderdaad: elk samengesteld

getal n heeft een kleinste positieve priemdeler p1, en uit de standaardontbinding van n volgt eenvoudig dat p21 ≤ n.

5Blijkbaar werd de hoofdstelling van de rekenkunde pas voor het eerst expliciet vermeld door Gauss in 1798, wellicht omdat hij de eerstewas die het belang hiervan inzag. Uiteraard was dit resultaat bekend bij eerdere wiskundigen, maar Gauss was de eerste die getaltheorieop een systematische manier ontwikkeld heeft.

6Het formele algoritme werd in 1977 ontworpen door Ron Rivest, Adi Shamir en Len Adleman (vandaar de afkorting RSA).

XVI-14

Page 26: Deel XVI Getaltheorie (Hoofdstukken 1 en 2) (recto-verso)

3 Modelvoorbeeld. Bepaal de standaardonbinding van 5566. Maak gebruik van je grafische rekenmachine.

Oplossing. Het getal 5566 is even, zodat het deelbaar is door 2. Zo vinden we al dat 5566 = 2 · 2783. We gaanna of 2783 deelbaar is door een positief priemgetal kleiner of gelijk aan

√2783 = 52, 7 . . .. Zo niet, dan is 2783

zelf een priemgetal.

Met behulp van de zeef van Eratosthenes vinden we de positieve priemgetallen kleiner dan 53 gelijk aan

2, 3, 5, 7, 11, 13, 17, 19, 23, 29, 31, 37, 41, 43, 47.

Alvast is 2783 niet deelbaar door 2, 3 en 5 (zie oefeningen 32 en 35). Met behulp van de rekenmachine gaan wena of 2783 deelbaar is door 7, 11, 13, 17, 19, 23, 29, 31, 37, 41, 43 en 47. Zo vinden we dat

5566 = 2 · 2783 = 2 · 11 · 253.

We herhalen dit proces met het deeltal 253. Omdat√

253 = 15, 9 . . . en we deelbaarheid door de positievepriemgetallen kleiner dan 11 al nagegaan hebben, hoeven we nu enkel de priemgetallen 11 en 13 te controleren.Zo vinden we uiteindelijk de standaardontbinding van 5566:

5566 = 2 · 11 · 253 = 2 · 11 · 11 · 23 = 2 · 112 · 23.

De hoofdstelling van de rekenkunde leidt tot een krachtig criterium voor deelbaarheid van twee gehele getallen. Zo isbijvoorbeeld

a = 57 · 193 · 372 deelbaar door b = 53 · 372

omdat elke priemdeler van b ook een priemdeler van a is en omdat de exponenten van de priemdelers van b kleiner ofgelijk zijn aan de exponenten van de priemdelers van a. Om ook de priemdeler 19 van a in rekening te brengen, is hethandig om de standaardontbindingen van a en b te herschrijven als

a = 57 · 193 · 372 en b = 53 · 190 · 372.

Zo kunnen voor elk tweetal gehele getallen verschillend van nul de standaardontbinding herschrijven met dezelfdepriemgetallen. Als bijvoorbeeld a = 3 · 52 en b = 2 · 73 dan kunnen we dit schrijven als

a = 20 · 3 · 52 · 70 en b = 2 · 30 · 50 · 73.Zo hoeven we niet steeds bij te houden welke priemfactoren wel of niet in a en b bevat zijn.

3 Gevolg. Zij a, b ∈ Z0 en stel

a = e pa11 p

a22 · · · pak

k en b = e′ pb11 pb22 · · · pbkk

waarbij e, e′ ∈ Z∗, k ∈ N, ai, bi ∈ N en p1 < p2 < . . . < pk positieve priemgetallen zijn. Dan geldt:

b | a ⇔ bi ≤ ai voor alle i

Bewijs. We hebben:

a

b=e pa1

1 pa22 · · · pak

k

e′ pb11 pb22 · · · pbkk

= e′′ pa1−b11 pa2−b2

2 · · · pak−bkk

waarbij e′′ ∈ Z∗. Stel eerst dat bi ≤ ai voor alle i. Dan geldt:

a1 − b1 ≥ 0 , a2 − b2 ≥ 0 , . . . , ak − bk ≥ 0

⇒ pa1−b11 ∈ Z , pa2−b2

2 ∈ Z , . . . , pak−bkk ∈ Z

⇒ pa1−b11 pa2−b2

2 · · · pak−bkk ∈ Z

⇒ a

b∈ Z

⇒ b | a.Omgekeerd, stel dat b | a. Dan is a = bq voor een zeker geheel getal q, dus

e pa11 p

a22 · · · pak

k = e′ pb11 pb22 · · · pbkk q.

Mocht bijvoorbeeld b1 > a1 dan zou

e pa22 · · · pak

k = e′ pb1−a11 pb22 · · · pbkk q.

Maar dan leidt het linkerlid tot een standaardontbinding die het priemgetal p1 niet bevat, terwijl het rechterlidleidt tot een standaardontbinding die het priemgetal p1 wel bevat. Dit is strijdig met de hoofdstelling van derekenkunde. We besluiten dat b1 ≤ a1. Analoog bewijst men dat bi ≤ ai voor alle i.

XVI-15

Page 27: Deel XVI Getaltheorie (Hoofdstukken 1 en 2) (recto-verso)

2.6 Grootste gemene delers

We hernemen een eerder voorbeeld: −12Z + 8Z = 4Z. Merk op dat ook −12Z + 8Z = (−4)Z. Niet toevallig zijn 4 en−4 zogenaamde grootste gemene delers van −12 en 8:

(1) 4 en −4 zijn beide delers van −12 en 8, en

(2) 4 en −4 zijn beide een veelvoud van elke andere deler c van −12 en 8.

In het algemeen worden de grootste gemene delers van a en b als volgt gedefinieerd.

3 Definitie. Zij a, b ∈ Z niet beide nul. Een grootste gemene deler van a en b is een geheel getal d 6= 0 dat voldoetaan de volgende twee voorwaarden:

(1) d | a en d | b,(2) ∀c ∈ Z0 : c | a en c | b⇒ c | d.

Is 1 een grootste gemene deler van a en b, dan noemen we a en b relatief priem.

Voorbeeld. Een grootste gemene deler van −18 en 24 is −6. Een grootste gemene deler van 108 en 0 is 108. Degetallen 38 en −7 zijn relatief priem.

3 Eigenschap 1. Zij a, b ∈ Z niet beide nul en d ∈ Z0. Dan zijn de volgende uitspraken equivalent:

(i) d is een grootste gemene deler van a en b,

(ii) dZ = aZ + bZ.

Bewijs. We bewijzen de twee implicaties (i)⇒(ii) en (ii)⇒(i).

(i)⇒(ii) Stel dat d een grootste gemene deler van a en b is. Wegens een eerder lemma bestaat er een geheel getald′ waarvoor aZ + bZ = d′Z. Merk op dat d′ 6= 0 (waarom?). We beweren dat d′ = ±d.

Enerzijds volgt uit aZ + bZ = d′Z dat d′ = ak+ bl voor zekere k, l ∈ Z. Omdat d | a en d | b, volgt nu dat d | d′.Anderzijds is a = a · 1 + b · 0 ∈ aZ + bZ. Omdat aZ + bZ = d′Z geldt dus dat a ∈ d′Z. Dus a = d′q voor eengeheel getal q, dus d′ | a. Analoog is d′ | b. Omdat d een grootste gemene deler van a en b is, volgt hieruit datd′ | d. Dus d = ±d′. Hieruit volgt dat dZ = d′Z zodat dZ = aZ + bZ.

(ii)⇒(i) Stel dat aZ+ bZ = dZ. Dan is a ∈ dZ zodat d | a. Analoog is d | b. Om aan te tonen dat d een grootstegemene deler van a en b is, nemen we een c ∈ Z0 zodat c | a en c | b. Omdat aZ + bZ = dZ is ak + bl = d voorzekere k, l ∈ Z. Hieruit volgt dat c | d.

Een belangrijk gevolg is dat elk tweetal gehele getallen a, b niet beide nul, precies twee grootste gemene delers hebben,waarvan de ene tegengesteld is aan de andere (ga na). De positieve grootste gemene deler van a en b zullen we metggd(a, b) noteren. Merk dus op dat

ggd(a, b)Z = aZ + bZ = (− ggd(a, b))Z.

Voorgaande resultaten leiden nu tot de zogenaamde stelling van Bachet-Bezout7.

Claude Gaspard Bachet deMeziriac(1581 - 1638)

3 Stelling (Bachet-Bezout). Zij a, b ∈ Z niet beide nul en zij d = ggd(a, b) depositieve grootste gemene deler van a en b. Dan geldt:

(1) er bestaan gehele getallen k, l waarvoor d = ak + bl,

(2) d is het kleinste positief geheel getal dat lineaire combinatie is van a en b,

(3) elke lineaire combinatie van a en b is een veelvoud van d.

Bewijs. (1) Uit d ∈ dZ = aZ + bZ volgt het bestaan van twee gehele getallenk, l waarvoor d = ak + bl.

(2) Uit (1) volgt dat d een lineaire combinatie is van a en b. Is d′ een anderpositief geheel getal dat een lineaire combinatie is van a en b, dan is d′ ∈aZ + bZ = dZ zodat d′ = dq voor een zekere q ∈ Z. Uit d′ > 0 volgt datq ≥ 1 en uit d′ 6= d volgt dat q > 1. Er volgt nu dat d′ = dq > d. Daarmeeis aangetoond dat d het kleinste positief geheel getal is dat een lineairecombinatie is van a en b.

(3) Zij x = am + bn een lineaire combinatie van a en b, waarbij m,n ∈ Z. Dan is x ∈ aZ + bZ = dZ, zodat xeen veelvoud van d is.

De schrijfwijze van grootste gemene deler als lineaire combinatie van a en b is niet uniek. Zo is bijvoorbeeld ggd(6, 4) = 2en 2 = 6 · 1 + 4 · (−1) = 6 · (−3) + 4 · 5.

7In 1779 bewees Etienne Bezout dit resultaat voor veeltermen. Het gelijkaardig resultaat voor gehele getallen kan gevonden worden inhet werk van Claude Gaspard Bachet de Meziriac (1624)

XVI-16

Page 28: Deel XVI Getaltheorie (Hoofdstukken 1 en 2) (recto-verso)

Kennen we van a, b ∈ Z0 de standaardontbinding, dan kunnen we ggd(a, b) op een eenvoudige manier vinden. Beschouwbijvoorbeeld

a = 11 016 = 23 · 34 · 17 · 190 en b = 21 888 = 27 · 32 · 170 · 19

dan zien we in dat de volgende formule geldt:

ggd(a, b) = 2min(3,7) · 3min(4,2) · 17min(1,0) · 19min(0,1) = 23 · 32 · 170 · 190 = 72.

Deze formule is eenvoudig in het algemeen te bewijzen. Deze eigenschap is erg handig om basiseigenschappen vangrootste gemene deler te bewijzen, zoals de handige formule ggd(na, nb) = n ggd(a, b) voor n ∈ N0, zie Oefening 64(c).

3 Eigenschap 2. Zij a, b ∈ Z0 en stel a = e pa11 p

a22 · · · pak

k en b = e′ pb11 pb22 · · · pbkk waarbij e, e′ ∈ Z∗, k ∈ N,

ai, bi ∈ N en p1 < p2 < . . . < pk positieve priemgetallen zijn. Dan geldt:

ggd(a, b) = pmin(a1,b1)1 p

min(a2,b2)2 · · · pmin(ak,bk)

k

Bewijs. Beschouw het geheel getal

d = pmin(a1,b1)1 p

min(a2,b2)2 · · · pmin(ak,bk)

k .

We zullen bewijzen dat d = ggd(a, b) door de definitie van grootste gemene deler na te gaan.

Omdat min(ai, bi) ≤ ai voor alle i is d | a. Analoog is d | b. Stel nu dat c ∈ Z0 zodat c | a en c | b. Dan kan destandaardontbinding van c geschreven worden als

c = e′′ pc11 pc22 · · · pckk

waarbij e′′ ∈ Z∗, ci ∈ N en ci ≤ ai en ci ≤ bi en dus ci ≤ min(ai, bi) voor alle i, zodat ook c | d.

3 Modelvoorbeeld 1. Zij a, b, c ∈ Z0 en stel dat ggd(a, b) = 1 en c | a. Bewijs dat ggd(b, c) = 1.

Oplossing. We herschrijven de standaardontbinding van a, b en c als

a = epa11 p

a22 . . . pak

k en b = e′pb11 pb22 . . . pbkk en c = e′′pc11 p

c22 . . . pckk

waarbij e, e′, e′′ ∈ Z∗, k ∈ N, ai, bi, ci ∈ N en p1 < p2 < . . . < pk positieve priemgetallen zijn. Uit ggd(a, b) = 1volgt dat min(ai, bi) = 0 en uit c | a volgt dat ci ≤ ai voor elke i. Dus voor elke i is ai = 0 of bi = 0. Als ai = 0dan is noodzakelijk ook ci = 0, zodat min(bi, ci) = 0. Als bi = 0 dan geldt logischerwijze ook dat min(bi, ci) = 0.Dus min(bi, ci) = 0 voor elke i, zodat ggd(b, c) = 1.

Voorlopig moeten we, om de positieve grootste gemene deler van a, b ∈ N0 te berekenen, eerst de standaardontbindingenvan a en b bepalen. We hebben reeds opgemerkt dat daar geen efficient algoritme voor bekend is. Er is echter eensimpele manier om ggd(a, b) te berekenen: het zogenaamde Euclidisch algoritme. In dat proces wordt ggd(a, b) herleidtot ggd(m,n) voor zekere m,n die kleiner zijn dan a en b. Deze werkwijze steunt op de volgende

3 Stelling. Zij a, b ∈ Z niet beide nul. Dan geldt voor elke k ∈ Z dat

ggd(a, b) = ggd(a, ak + b)

Bewijs. We zullen aantonen dat ggd(a, b) | ggd(a, ak + b) en ggd(a, ak + b) | ggd(a, b). Omdat ggd(a, b) enggd(a, ak + b) beide natuurlijke getallen zijn, volgt hier dan uit dat ggd(a, b) = ggd(a, ak + b).

Enerzijds is ggd(a, b) | a en ggd(a, b) | b dus ggd(a, b) | (ak + b) zodat ggd(a, b) | ggd(a, ak + b). Anderzijds isggd(a, ak+ b) | a en ggd(a, ak+ b) | (ak+ b) zodat ggd(a, ak+ b) | (ak+ b−ak) waaruit ggd(a, ak+ b) | ggd(a, b).

In het bijzonder is ggd(a, b) = ggd(a, b − a) en ggd(a, b) = ggd(a − b, b). Door dit herhaaldelijk toe te passen, kanhet berekenen van ggd(a, b) herleid worden tot het berekenen van ggd(d, 0) voor een zeker geheel getal d. Dit proceswordt versneld door de deling met rest van het grootste getal door het kleinste getal uit te voeren.

3 Modelvoorbeeld 2. Bepaal ggd(195, 286) en schrijf als een lineaire combinatie van 195 en 286.

Oplossing. We hebben:

ggd(195, 286) = ggd(195, 286− 195) = ggd(195, 91)

= ggd(195− 2 · 91, 91) = ggd(13, 91)

= ggd(13, 91− 7 · 13) = ggd(13, 0) = 13

waarbij de laatste gelijkheid volgt uit het feit dat 13 het grootste geheel getal is dat zowel 13 als 0 deelt. Doorop onze stappen terug te keren, kunnen we 13 schrijven als een lineaire combinatie van 195 en 286:

13 = 195− 2 · 91

= 195− 2 · (286− 195)

= 3 · 195 + (−2) · 286.

XVI-17

Page 29: Deel XVI Getaltheorie (Hoofdstukken 1 en 2) (recto-verso)

2.7 Kleinste gemene veelvouden

3 Definitie. Zij a, b ∈ Z0. Een kleinste gemene veelvoud van a en b is een geheel getal v 6= 0 dat voldoet aan devolgende twee voorwaarden:

(1) a | v en b | v,

(2) ∀c ∈ Z0 : a | c en b | c⇒ v | c.Voorbeeld. Een kleinste gemene veelvoud van −18 en 24 is 72. Een kleinste gemene veelvoud van 17 en 35 is 595.

3 Eigenschap 1. Zij a, b ∈ Z0 en v ∈ Z0. Dan zijn de volgende uitspraken equivalent:

(i) v is een kleinste gemene veelvoud van a en b,

(ii) vZ = aZ ∩ bZ.

Bewijs. We bewijzen de twee implicaties (i)⇒(ii) en (ii)⇒(i).

(i)⇒(ii) Stel dat v een kleinste gemene veelvoud van a en b is. Wegens een eerder lemma bestaat er een geheelgetal v′ waarvoor aZ ∩ bZ = v′Z. Merk op dat v′ 6= 0 (waarom?). We beweren dat v′ = ±v.

Enerzijds volgt uit aZ ∩ bZ = v′Z dan v′ = ak = bl voor zekere k, l ∈ Z. Dus a | v′ en b | v′. Omdat v eenkleinste gemene veelvoud van a en b is, volgt nu dat v | v′.Anderzijds volgt uit a | v en b | v dat v ∈ aZ ∩ bZ = v′Z, zodat ook v′ | v. Dus v′ = ±v. Hieruit volgt datvZ = v′Z, zodat vZ = aZ ∩ bZ.

(ii)⇒(i) Stel dat aZ ∩ bZ = vZ. Omdat v ∈ vZ is v ∈ aZ zodat a | v. Analoog is b | v. Om aan te tonen datv een kleinste gemene veelvoud van a en b is, nemen we een c ∈ Z0 zodat a | c en b | c. Dus dan is c ∈ aZ enc ∈ bZ zodat c ∈ aZ ∩ bZ = vZ. Hieruit volgt dat v | c.

Een belangrijk gevolg is dat elk tweetal gehele getallen a, b beide verschillend van nul, precies twee kleinste gemeneveelvouden hebben, waarvan de ene tegengesteld is aan de andere (ga na). Het positieve kleinste gemene veelvoud vana en b zullen we met kgv(a, b) noteren. Merk dus op dat

kgv(a, b)Z = aZ ∩ bZ = (− kgv(a, b))Z.

Het bewijs van de volgende eigenschap laten we als Oefening 79 voor de lezer.

3 Eigenschap 2. Zij a, b ∈ Z0 en stel a = e pa11 p

a22 . . . pak

k en b = e′ pb11 pb22 . . . pbkk waarbij e, e′ ∈ Z∗, k ∈ N,

ai, bi ∈ N en p1 < p2 < . . . < pk positieve priemgetallen zijn. Dan geldt:

kgv(a, b) = pmax(a1,b1)1 p

max(a2,b2)2 . . . p

max(ak,bk)k

Kennen we van twee gehele getallen hun standaardontbinding niet, dan kan de volgende stelling aangewend wordenom hun kleinste gemene veelvoud te berekenen.

3 Stelling. Zij a, b ∈ N0. Dan geldt:

ggd(a, b) · kgv(a, b) = ab

Bewijs. Herschrijven we de standaardontbinding van a en b als

a = pa11 p

a22 . . . pak

k en b = pb11 pb22 . . . pbkk

waarbij k ∈ N, ai, bi ∈ N en p1 < p2 < . . . < pk positieve priemgetallen zijn, dan is

ggd(a, b) · kgv(a, b) = pmin(a1,b1)+max(a1,b1)1 p

min(a2,b2)+max(a2,b2)2 . . . p

min(ak,bk)+max(ak,bk)k .

Om de stelling aan te tonen, volstaat het te bewijzen dat min(ai, bi)+max(ai, bi) = ai + bi voor elke i. Daarvooronderscheiden we twee gevallen. Als ai ≤ bi dan is min(ai, bi) = ai en max(ai, bi) = bi zodat min(ai, bi) +max(ai, bi) = ai+bi. Als ai > bi dan is min(ai, bi) = bi en max(ai, bi) = ai zodat min(ai, bi)+max(ai, bi) = ai+bi.

3 Modelvoorbeeld. Bepaal het kleinste gemene veelvoud van 195 en 286.

Oplossing. Met behulp van het Euclidisch algoritme vinden we dat ggd(195, 286) = 13, zie Modelvoorbeeld 2 uitde vorige paragraaf. Wegens de bovenstaande stelling is dan

kgv(195, 286) =195 · 286

ggd(195, 286)=

55 770

13= 4290.

XVI-18

Page 30: Deel XVI Getaltheorie (Hoofdstukken 1 en 2) (recto-verso)

2.8 Toepassingen

Priemgetallen zijn onlosmakelijk verbonden met de manier waarop gegegevens beveiligd worden. Heel wat belangrijkecodeermechanismen steunen op de kennis van zeer grote priemgetallen. Tot op heden is er geen algoritme bekenddat priemgetallen op een efficiente manier kan voortbrengen. Toch zijn wiskundigen al eeuwenlang op zoek geweestnaar formules om priemgetallen te genereren. Zo vond Leonhard Euler in 1772 dat n2 + n+ 41 een priemgetal is voorn = 0, 1, 2, . . . , 39. En in 1837 bewees Johann Dirichlet dat er in elke rekenkundige rij a, a+ v, a+ 2v, a+ 3v, . . . meta, v ∈ Z0 relatief priem, oneindig veel priemgetallen voorkomen.

Hierna beschrijven we priemgetallen van de vorm 2n−1 en 2n + 1. In een laatste toepassing laten we zien hoe kleinstegemene veelvoud en grootste gemene deler een rol spelen bij de werking van tandwielen.

Toepassing 1 - Fermatgetallen

Fermatgetallen zijn vernoemd naar Pierre de Fermat, die deze getallen voor het eerst bestudeerde.

Pierre de Fermat(1601-1665)

3 Definitie Zij n ∈ N. Het n-de Fermatgetal is gelijk aan

Fn = 22n

+ 1

3 Voorbeeld. De eerste zeven Fermatgetallen zijn gelijk aan:

F0 = 220

+ 1 = 3

F1 = 221

+ 1 = 5

F2 = 222

+ 1 = 17

F3 = 223

+ 1 = 257

F4 = 224

+ 1 = 65 537

F5 = 225

+ 1 = 4 294 967 297

F6 = 226

+ 1

F6 = 18 446 744 073 709 551 617.

In 1650 vermoedde Fermat dat elk Fermatgetal een priemgetal is, maar gaf toe dathij dit niet kon aantonen. Zo zijn inderdaad de eerste vijf Fermatgetallen alle priem.Het vermoeden van Fermat werd weerlegd door Leonhard Euler 1732, die de stan-daardontbinding van het Fermatgetal F5 heeft berekend:

F5 = 641 · 6 700 417.

Intussen is aangetoond dat ook F6, F7, . . . , F32 geen priemgetallen zijn. Het is onbekend of er nog andere Fermat-priemgetallen zijn. Sommigen vermoeden dat het aantal Fermatpriemgetallen eindig is. In dat geval zou het aantalpriemgetallen van de vorm 2n + 1 ook eindig zijn. Dit volgt uit de volgende eigenschap.

3 Eigenschap. Zij n ∈ N0. Als 2n + 1 een priemgetal is, dan is n een macht van 2.

Bewijs. Beschouw de veeltermidentiteiten (zie ook Oefening 10(b) uit Hoofdstuk 1):

x3 + 1 = (x+ 1)(x2 − x+ 1)

x5 + 1 = (x+ 1)(x4 − x3 + x2 − x+ 1)

x7 + 1 = (x+ 1)(x6 − x5 + x4 − x3 + x2 − x+ 1)

...

xk + 1 = (x+ 1)(xk−1 − xk−2 + xk−3 − · · ·+ x2 − x+ 1) waarbij k > 1 oneven is.

Mocht n = 3q met q ∈ Z dan volgt uit de eerste identiteit dat

an + 1 = a3q + 1 = (aq)3 + 1 = (aq + 1)(a2q − aq + 1)

zodat an + 1 geen priemgetal is, een strijdigheid.

Mocht n = 5q met q ∈ Z dan volgt uit de tweede identiteit dat

an + 1 = a5q + 1 = (aq)5 + 1 = (aq + 1)(a4q − a3q + a2q − aq + 1)

zodat an + 1 geen priemgetal is, een strijdigheid.

In het algemeen, mocht n = kq met q, k ∈ Z en k > 1 oneven, dan volgt uit de laatste identiteit dat

an + 1 = akq + 1 = (aq)k + 1 = (aq + 1)(a(k−1)q − a(k−2)q + a(k−3)q − · · ·+ a2q − aq + 1)

zodat an + 1 geen priemgetal is, een strijdigheid. We besluiten dat n geen oneven deler groter dan 1 heeft, zodatn een macht van 2 is.

XVI-19

Page 31: Deel XVI Getaltheorie (Hoofdstukken 1 en 2) (recto-verso)

Fermatgetallen zijn om verschillende redenen belangrijk. Zo heeft Carl Friedrich Gauss ingezien dat een regelmatigen-hoek met passer en liniaal kan worden geconstrueerd als en slechts als n een product is van een macht van 2 meteen aantal verschillende Fermatpriemen. De bewering van Gauss werd bewezen door Pierre Wantzel 1837.

In de volgende stelling vermelden we een van de belangrijkste eigenschappen van de rij van Fermatgetallen: eenrecursief voorschrift. Een gevolg hiervan is dat twee verschillende Fermatgetallen relatief priem zijn. Dit resultaatwerd door Christian Goldbach in 1730 in een brief aan Leonhard Euler neergeschreven.

3 Stelling (Goldbach). Beschouw de rij van Fermatgetallen F0, F1, F2, F3, F4, F5, . . ..

(a) Voor elke n ∈ N0 geldt: Fn = 2 + F0 · F1 · F2 · · ·Fn−1.

(b) Zij Fm, Fn twee verschillende Fermatgetallen. Dan is ggd(Fm, Fn) = 1.

Bewijs. (a) We bewijzen de formule Fn = 2 + F0 · F1 · F2 · · ·Fn−1 met inductie op n.

(i) Inductiebasis. Voor n = 1 is enerzijds Fn = 5 en anderzijds is (vul aan):

2 + F0 · F1 · F2 · · ·Fn−1 = 2 + F0 = 2 + 3 = 5.

Dus voor n = 1 is de formule aangetoond.

(ii) Stel dat de formule waar is voor n = k (met k ≥ 1), dus dat

Fk = 2 + F0 · F1 · F2 · · ·Fk−1.

We moeten aantonen dat de formule waar is voor n = k + 1, dus we moeten aantonen dat

Fk+1 = 2 + F0 · F1 · F2 · · ·Fk.

Welnu,

2 + F0 · F1 · F2 · · ·Fk = 2 + F0 · F1 · F2 · · ·Fk−1 · Fk

= 2 + (Fk − 2) · Fk

= 2 + (22k − 1)(22

k

+ 1)

= 2 + 22k+1 − 1

= 22k+1

+ 1

= Fk+1.

Uit (i) en (ii) volgt nu dat de formule geldt voor alle n ≥ 1.

(b) Zonder de algemeenheid te schaden mogen we aannemen dat m < n. Uit (a) volgt dat

Fm | Fn − 2.

zodat ggd(Fm, Fn) | 2. Omdat elk Fermatgetal oneven is (waarom?) volgt hieruit dat ggd(Fm, Fn) = 1.

Adolf Hurwitz(1859-1919)

Omdat de rij van Fermatgetallen oneindig is, vinden we op deze manier een ander be-wijs voor de tweede stelling van Euclides: er zijn oneindig veel priemgetallen. Vreemdgenoeg heeft het tot 1891 geduurd vooraleer iemand dit voor de hand liggende gevolgvan de stelling van Goldbach gevonden heeft.

3 Stelling (tweede stelling van Euclides). Er zijn oneindig veel priemgetallen.

Bewijs van Hurwitz. Voor elke n ∈ N nemen we een priemdeler pn van het n-deFermatgetal Fn. Wegens de vorige stelling deel (b) zijn deze priemdelers ver-schillend van elkaar, zodat p0, p1, p2, . . . een niet-eindigende rij van verschillendepriemgetallen is. Hieruit volgt dat de rij van alle priemgetallen eveneens niet-eindigend is.

XVI-20

Page 32: Deel XVI Getaltheorie (Hoofdstukken 1 en 2) (recto-verso)

Toepassing 2 - Mersennegetallen

Mersennegetallen zijn vernoemd naar Marin Mersenne, die een formule probeerde te vinden om priemgetallen tegenereren.

Marin Mersenne(1588-1648)

3 Definitie Zij n ∈ N0. Het n-de Mersennegetal is gelijk aan

Mn = 2n − 1

3 Voorbeeld. De eerste zeven Mersennegetallen zijn gelijk aan:

M1 = 21 − 1 = 1

M2 = 22 − 1 = 3

M3 = 23 − 1 = 7

M4 = 24 − 1 = 15

M5 = 25 − 1 = 31

M6 = 26 − 1 = 63

M7 = 27 − 1 = 127.

Enkele voorbeelden doen ons vermoeden dat er een verband is tussen het al of niet priem zijn van een MersennegetalMn en het al of niet priem zijn van n.

3 Eigenschap. Zij n ∈ N0. Als Mn = 2n − 1 een priemgetal is, dan is n een priemgetal.

Bewijs. Stel dat Mn = 2n − 1 een priemgetal is en stel uit het ongerijmde dat n geen priemgetal is. Dan isn = ab voor zekere gehele getallen a, b met 1 < a < n en 1 < b < n. Uit

2n − 1 = 2ab − 1 = (2a − 1)(2(b−1)a + 2(b−2)a + 2(b−3)a + · · ·+ 22a + 2a + 1)

volgt dat Mn geen priemgetal is, een strijdigheid.

Het omgekeerde van de vorige eigenschap is niet waar: als n priem is dan hoeft Mn niet priem te zijn. Het kleinstetegenvoorbeeld is het Mersennegetal

M11 = 211 − 1 = 2047 = 23 · 89.

In 1644 beweerde Mersenne dat Mn een priemgetal is voor n = 2, 3, 5, 7, 13, 17, 19, 31, 67, 127, 257 en een samengesteldgetal is voor de andere waarden voor n met 1 ≤ n ≤ 257. De eerste fout in deze bewering werd pas in 1883 gevonden,toen Ivan Mikheevich Pervushin aantoonde dat M61 een priemgetal is. Vervolgens werden nog vier andere fouten inde bewering van Mersenne gevonden. In 1876 vond Eduard Lucas een manier om te testen of Mp een priemgetal is engebruikte deze methode om aan te tonen dat M127 priem is. De Mersenne priemgetallen Mn met 1 ≤ n ≤ 257 wordengegeven door

n = 2, 3, 5, 7, 13, 17, 19, 31, 61, 89, 107, 127.

de Arithmometer, eenmechanische rekenmachine

uit 1914

Het Mersennegetal

M127 = 170 141 183 460 469 231 731 687 303 715 884 105 727

telt 39 cijfers en bleef het grootst gekende priemgetal tot Ferrier in 1951 met behulpvan een mechanische rekenmachine vond dat

F =1

17

(2148 + 1

)= 20 988 936 657 440 586 486 151 264 256 610 222 593 863 921

een priemgetal is met 44 cijfers. Dit is meteen het grootste priemgetal dat zonderbehulp van een elektronische rekenmachine of computer werd gevonden. Met de komstvan de computers werden in het zelfde jaar al gauw grotere priemgetallen gevonden.Intussen werden 48 Mersenne priemgetallen gevonden, de grootste is M57 885 161 heeft17 425 170 cijfers (gevonden op 25 januari 2013). Het is meteen ook het grootstepriemgetal dat op moment van schrijven (juli 2015) bekend is8

8 In 1996 werd de GIMPS (Great Internet Mersenne Prime Search) opgericht, een samenwerkingsproject van duizenden vrijwilligersdie vrij beschikbare software gebruiken om nieuwe Mersenne Priemgetallen op te sporen. Het project heeft tot op april 2014 al veertienMersenne priemgetallen ontdekt, waarvan twaalf op het moment van hun ontdekking het grootst bekende priemgetal waren. Meewerkenkan via http://www.mersenne.org/ . Een actuele lijst met de tot op heden bekende Mersenne priemgetallen is terug te vinden ophttp://www.mersenne.org/primes/ .

XVI-21

Page 33: Deel XVI Getaltheorie (Hoofdstukken 1 en 2) (recto-verso)

Toepassing 3 - Tandwielen

Voor een draaiend tandwiel noteren we met r de straal (meter), z het aantal tanden (per omwenteling) en n hettoerental (omwentelingen per minuut). Dan is de omtrek van het tandwiel 2πr, het aantal tanden per meter gelijkaan z/(2πr), en in een willekeurig vast punt op de rand (bijvoorbeeld de onderkant van het tandwiel) passeren er perminuut zn tanden.

draaiende tandwielenmet overbrengings-

verhouding i = 40/16 = 2, 5

Beschouw nu twee tandwielen die in elkaar rollen (zie figuur). Willen de tandwielenniet slippen of blokkeren, dan moet het aantal tanden per meter op het eerste tandwielgelijk zijn aan het aantal tanden per meter op het eerste tandwiel9:

z12πr1

=z2

2πr2.

Verder zullen er, op de plaats waar de twee tandwielen in elkaar schuiven, per minuutevenveel tanden van het eerste tandwiel als van het tweede tandwiel passeren, zodat

z1 · n1 = z2 · n2.

Hieruit besluiten we datn1n2

=z2z1

=r2r1

Deze verhouding noemt men de overbrengingsverhouding (of transmissiecijfer ofdrijfwerkverhouding) van het eerste naar het tweede tandwiel, en wordt met i ge-noteerd. Is bijvoorbeeld i > 1, dan zal de as van het tweede tandwiel langzamerdraaien dan de as van het eerste tandwiel, maar het tweede tandwiel levert wel eengroter krachtmoment als gevolg van de grotere straal.

Het volgende modelvoorbeeld maakt een link tussen tandwielen en de begrippen grootste gemene deler en kleinstegemene veelvoud duidelijk.

3 Modelvoorbeeld. Twee tandwielen werken in op elkaar, het eerste heeft 64 tanden en doet 18 omwentelingenper minuut, het tweede tandwiel heeft 24 tanden. De straal van het tweede tandwiel is 10 cm.

(a) Bepaal het toerental van het tweede tandwiel.

(b) Bepaal de snelheid van een willekeurige tand op het eerste tandwiel en de snelheid van een willekeurigetand op het tweede tandwiel.

(c) Na hoeveel omwentelingen van het kleinste tandwiel staan beide tandwielen voor de eerste keer weer inprecies dezelfde stand? Maak gebruik van je grafische rekenmachine.

Oplossing.

(a) Noemen we het toerental van het tweede tandwiel n2, dan volgt uit de overbrengingsverhouding

i =24

64=

18

n2

dat n2 = 48. Het toerental van het tweede tandwiel bedraagt dus 48 omwentelingen per minuut.

(b) Per omwenteling legt een tand op het tweede tandwiel een afstand van 2π ·0, 1 = 0, 2π meter af. De snelheidvan een willekeurige tand op het tweede tandwiel is dus gelijk aan

v = 0, 2π meter/omwenteling× 48 omwentelingen/minuut = 9, 6π = 30, 15 . . . meter/minuut.

Op de plaats waar de twee tandwielen in elkaar schuiven, zullen per minuut dan evenveel tanden van heteerste tandwiel als van het tweede tandwiel passeren. Daarom is de snelheid van een willekeurige tand ophet eerste tandwiel ook gelijk aan v.

(c) Noem k het aantal tanden waarover we het eerste tandwiel moeten verschuiven. Wil het eerste tandwiel indezelfde stand staan, dan moet k ∈ 64Z. Over een tand in het eerste tandwiel verschuiven betekent ookdat we over een tand in het tweede tandwiel draaien. Wil ook het tweede tandwiel na k tanden in dezelfdestand staan, dan moet k ∈ 24Z. Zo vinden we dat

k ∈ 64Z ∩ 24Z = kgv(64, 24)Z = 192Z.

Het kleinste aantal tanden dat we het eerste (of het tweede) tandwiel moeten verschuiven is dus gelijk aan192. Het tweede tandwiel telt minder tanden en is dus kleiner dan het eerste tandwiel. We besluiten dathet kleinste tandwiel 192/24 = 8 omwentelingen moet maken.

9Strikt genomen zijn de diameters 2r1 en 2r2 van de tandwielen niet de buitendiameters, maar wel de diameters van de raakcirkels.

XVI-22

Page 34: Deel XVI Getaltheorie (Hoofdstukken 1 en 2) (recto-verso)
Page 35: Deel XVI Getaltheorie (Hoofdstukken 1 en 2) (recto-verso)

Oefeningen

2 Deelbaarheid van gehele getallen Basis Verdieping Uitbreiding? ?? ? ?? ? ??

2.1 Deelbaarheid en delers 12

34

5 67

8 9

2.2 Deling met rest 10111213

1415

1617

181920

2122

2.3 Veelvouden en lineaire combinaties 232425

2627

28 29 30 31 323334

35 3637

2.4 Eenheden, priemgetallen, samengestelde getallen 3839

404142

43 44 45

2.5 Hoofdstelling van de rekenkunde 4647

4849

50 51 525354

55 56

2.6 Grootste gemene delers 575859

6061626364

636465

666768

6970

7172

2.7 Kleinste gemene veelvouden 73 7475

76 77 78 79

2.8 Toepassingen 808182

8384

85 86 8788

89 90

Oefeningen bij §2.1

B Oefening 1. Bepaal alle gehele getallen n die een deler zijn van 5n− 12.

B Oefening 2. Waar of vals? Beoordeel de volgende uitspraken. Verklaar telkens je antwoord.

(a) Elk geheel getal heeft minstens vier delers.

(b) Elk geheel getal heeft minstens een echte deler.

(c) Elk geheel getal heeft een eindig aantal delers.

(d) Voor elk geheel getal a geldt: del(a) = {b | b ∈ Z en b | a}.

(e) Voor elk geheel getal a geldt: a ∈ del(a).

B? Oefening 3. Bewijs de volgende uitspraken.

(a) Zij a, b ∈ Z met b 6= 0. Dan geldt: b | a⇔ ∃! q ∈ Z : qb = a.

(b) Zij d, d′ ∈ Z0 en stel dat d | d′ en d′ | d. Dan is d = d′ of d = −d′.

B? Oefening 4 (Vlaamse Wiskunde Olympiade 1991 eerste ronde).Het geheel getal q waarvoor 5050 = 2525q is gelijk aan

(A) 2525 (B) 1025 (C) 10025 (D) 225 (E) 2 · 2525

B?? Oefening 5.

(a) Stel dat a ∈ Z en dat del(a) = del(10). Waaraan kan a dan gelijk zijn?

(b) Formuleer en bewijs een veralgemening van (a) voor willekeurige gehele getallen.

V Oefening 6. Toon aan dat 20153 − 19773 deelbaar is door 19.

V Oefening 7. Zij n een oneven natuurlijk getal. Bewijs dat 8 | n2 − 1.

XVI-23

Page 36: Deel XVI Getaltheorie (Hoofdstukken 1 en 2) (recto-verso)

V? Oefening 8. Zij a ∈ Z en n ∈ N0. Bewijs dat a− 1 een deler is van an − 1.

U Oefening 9 (basiseigenschappen van deelbaarheid). Zij a, b, c, k, l ∈ Z en d, e ∈ Z0. Bewijs volgende uitspraken.

(i) 1 | a en −1 | a

(ii) a 6= 0⇒ a | 0, a | a en −a | a

(iv) d | a⇒ d | ka en ed | ea

(vi) d | a en d | b⇒ d | ab

(vii) d | e en e | a⇒ d | a

Oefeningen bij §2.2

B Oefening 10. Bepaal telkens het quotient en de rest bij deling van a door b.

(a) a = 60 en b = 7

(b) a = −60 en b = 7

(c) a = −34 en b = 5

(d) a = 27 en b = −8

(e) a = −64 en b = −8

(f) a = 15 en b = −28

B Oefening 11. Bepaal alle gehele getallen die het deeltal zijn van een deling door 5, die 18 als quotient opleveren.

B Oefening 12. Het quotient van een opgaande deling van twee gehele getallen is −24. Als men het deeltal verdubbelten 5 bij de deler optelt, dan gaat de deling nog steeds op en is het quotient van deling gelijk aan 60. Welke zijn dedeler en het deeltal?

B Oefening 13. De deling van een geheel getal door 73 heeft als rest 3. Deelt men hetzelfde geheel getal door 70, danis de rest 48. De quotienten van beide delingen zijn gelijk. Bepaal dat geheel getal.

B? Oefening 14. Maak de deling met rest van 284 door 23, en bepaal het grootste geheel getal dat men bij het deeltalmag optellen of aftrekken zonder dat het quotient bij deling door 23 verandert.

B? Oefening 15. Van een deling met rest is de deler gelijk aan −2136 en het quotient gelijk aan 49.

(a) Bepaal het deeltal waarvoor de rest bij deling van deler door deeltal zo groot mogelijk is.

(b) Bepaal het deeltal waarvoor de rest bij deling van deler door deeltal zo klein mogelijk is.

B?? Oefening 16. Als twee gehele getallen bij deling door hun verschil dezelfde resten overlaten, dan verschillen dequotienten met 1. Bewijs.

B?? Oefening 17. Een deling laat een rest 7 over. Telt men 1 op bij het deeltal, dan wordt het quotient 1 groter. Bepaalalle mogelijkeden voor de deler.

V Oefening 18. Men deelt een geheel getal door 4, de rest is gelijk aan 2. Als men dat getal door 6 deelt, dan is derest nooit 3. Bewijs.

V Oefening 19. Een deling door 58 laat 40 tot rest over. Tussen welke getallen ligt het natuurlijk getal, dat men vanhet deeltal moet aftrekken, om het quotient met 1 te verkleinen?

V Oefening 20 (Vlaamse Wiskunde Olympiade 1987 eerste ronde).a, n ∈ N0 en an geeft bij deling door 73 rest 2, an+1 geeft bij deling door 73 rest 69. Voor de rest r bij deling van adoor 73 geldt

(A) 0 ≤ r < 10 (B) 10 ≤ r < 30 (C) 30 ≤ r < 50 (D) 50 ≤ r < 70 (E) r ≥ 70

V? Oefening 21. Bepaal twee gehele getallen waarvan de som 266 is en hun quotient bij deling met rest gelijk is aan 13.Geef nadien alle mogelijkheden voor die gehele getallen.

V? Oefening 22. Zij a en b twee gehele getallen, noem q en r het quotient en r de rest bij deling van a door b en steldat q 6= 0. Bewijs dat het quotient bij deling van a door q gelijk is aan de som van b en het quotient bij deling van rdoor q.

XVI-24

Page 37: Deel XVI Getaltheorie (Hoofdstukken 1 en 2) (recto-verso)

Oefeningen bij §2.3

B Oefening 23. Waar of vals? Beoordeel de volgende uitspraken. Verklaar telkens je antwoord.

(a) Er bestaat een geheel getal a waarvoor de verzameling aZ een eindig aantal elementen heeft.

(b) Zij a, b ∈ Z waarvoor aZ = bZ. Dan is a = b.

B Oefening 24. Bewijs de volgende uitspraken.

(a) Zij a ∈ Z waarvoor 1 ∈ aZ. Dan is a = 1 of a = −1.

(b) Zij a ∈ Z. Dan geldt: aZ = {b ∈ Z | a is een deler van b}.

B Oefening 25. Bewijs dat het verschil van twee opeenvolgende kwadraten altijd een oneven getal is.

B? Oefening 26 (Vlaamse Wiskunde Olympiade 1987 eerste ronde).Welke van de volgende vijf uitspraken is waar?

(A) Het kwadraat van een oneven getal is soms even.

(B) Als x even is, zijn x en 2x twee opeenvolgende even getallen.

(C) Als x even is, is (x− 1)(x+ 1) oneven.

(D) Als x even is, is 107x soms oneven.

(E) Als x en y oneven zijn, is 3(x+ y) oneven.

B? Oefening 27. Bewijs dat het verschil van twee opeenvolgende derdemachten altijd een oneven getal is.

B?? Oefening 28. Zij a, b ∈ Z. Toon aan:a = ±b ⇔ aZ = bZ.

V Oefening 29. Zij a een oneven geheel getal. Toon aan dat a2 − 1 ∈ 8Z.

V? Oefening 30. Men deelt 18 door een veelvoud van 6, dat geen veelvoud van 18 is. Welke kan de rest zijn? Bewijs jeantwoord.

V?? Oefening 31. Zij n ∈ N. Bewijs dat 32n+1 + 2n+2 een veelvoud is van 7.

U Oefening 32 (kenmerk van deelbaarheid door 2 en door 5). Bewijs dat een natuurlijk getal deelbaar is door2 als en slechts als het cijfer van de eenheden deelbaar is door 2, en deelbaar is door 5 als en slechts als het cijfer vande eenheden deelbaar is door 5.

U Oefening 33 (kenmerk van deelbaarheid door 4 en door 25). Bewijs dat een natuurlijk getal deelbaar is door4 als en slechts als het getal gevormd door de cijfers van de eenheden en tientallen deelbaar is door 4, en deelbaar isdoor 25 als en slechts als het getal gevormd door de cijfers van de eenheden en tientallen deelbaar is door 25.

U Oefening 34 (kenmerk van deelbaarheid door 8 en door 125). Bewijs dat een natuurlijk getal deelbaar is door8 als en slechts als het getal gevormd door de cijfers van de eenheden, tientallen en honderdtallen deelbaar is door 8,en deelbaar is door 125 als en slechts als het getal gevormd door de cijfers van de eenheden, tientallen en honderdtallendeelbaar is door 125.

U? Oefening 35 (kenmerk van deelbaarheid door 3 en door 9). Bewijs dat een natuurlijk getal deelbaar is door3 als en slechts als de som van zijn cijfers deelbaar is door 3, en deelbaar is door 9 als en slechts als de som van zijncijfers deelbaar is door 9.Aanwijzing. Maak gebruik van Oefening 8 om aan te tonen dat 10n − 1 deelbaar is door 9 voor elke n ∈ N0.

U?? Oefening 36 (lemma 2). Zij a, b ∈ Z. Bewijs dat er een geheel getal v is waarvoor aZ ∩ bZ = vZ.

U?? Oefening 37 (idealen van Z). Zij I een niet-lege deelverzameling van Z. Dan is I een ideaal van Z, notatie I CZ,als aan de volgende twee voorwaarden is voldaan:

(1) voor alle r, s ∈ I geldt dat r − s ∈ I,

(2) voor alle k ∈ Z en r ∈ I geldt dat kr ∈ I.

Bewijs dat voor elke deelverzameling I ⊆ Z geldt:

I CZ ⇔ ∃d ∈ Z : I = dZ.

XVI-25

Page 38: Deel XVI Getaltheorie (Hoofdstukken 1 en 2) (recto-verso)

Oefeningen bij §2.4

B Oefening 38. Geef de zeven kleinste natuurlijke getallen die samengesteld zijn.

B Oefening 39. Zij a, p ∈ Z waarbij p een priemgetal is. Bepaal alle gehele getallen n die een deler zijn van an+ p.

B? Oefening 40. Bepaal alle positieve priemgetallen kleiner dan 100.

B? Oefening 41. Bepaal alle gehele getallen a en b waarvoor a2 − b2 = 79.

B? Oefening 42. Bepaal telkens alle natuurlijke getallen n waarvoor p een priemgetal is.

(a) p = 17n

(b) p = n2 − 1

(c) p = n2 − 4

(d) p = n3 + 1

V Oefening 43 (Vlaamse Wiskunde Olympiade 1990 eerste ronde).Definieer n! = 1 ·2 ·3 · · · (n−1) ·n, het produkt van de natuurlijke getallen van 1 tot en met n. Het aantal priemgetallenp met eigenschap

77! + 1 < p < 77! + 77

is gelijk aan

(A) 0 (B) 1 (C) 7 (D) 11 (E) 17

V? Oefening 44. Zij n ∈ N. Bewijs dat het verschil van twee opeenvolgende n-de machten altijd een oneven getal is.

U? Oefening 45 (bovengrens voor positieve priemdelers in samengestelde getallen).Zij n een positief samengesteld getal en noem p de kleinst positieve deler van n verschillend van 1. Bewijs:

(a) p is een priemgetal,

(b) p ≤ √n.

Oefeningen bij §2.5

B Oefening 46. Bepaal het grootste negatief geheel getal dat precies vijf verschillende priemdelers heeft.

B Oefening 47. Bepaal het kleinste positief geheel getal n waarvoor 1260n de derdemacht van een natuurlijk getal is.

B? Oefening 48. Bepaal telkens de standaardontbinding van het geheel getal. Maak gebruik van je grafische rekenma-chine.

(a) −899

(b) 1485

(c) −5820

(d) 9075

B? Oefening 49 (Vlaamse Wiskunde Olympiade 1986 eerste ronde).Hoeveel verschillende positieve delers heeft het getal 30 030 = 2 · 3 · 5 · 7 · 11 · 13?

(A) 6 (B) 36 (C) 62 (D) 64 (E) 128

V Oefening 50. Bepaal de rest bij deling van 2015! door 2016.

V? Oefening 51. Bepaal 100 opeenvolgende gehele getallen waarvan geen enkele een priemgetal is.

U Oefening 52 (perfecte getallen). Een natuurlijk getal wordt perfect (of volmaakt)10 genoemd als het gelijk is aande som van zijn echte positieve delers. Toon aan dat 6 en 28 perfecte getallen zijn.

U Oefening 53 (gehele getallen met zes delers). Toon aan dat de gehele getallen met zes verschillende delersgegeven worden door

±p2 waarbij p een priemgetal is.

U Oefening 54 (bevriende getallen). Van twee verschillende natuurlijke getallen a, b wordt gezegd dat ze bevriend11

zijn als de som van de echte positieve delers van a gelijk is aan b, en de som van de echte positieve delers van b gelijkis aan a. Toon aan dat 220 en 284 bevriende getallen zijn.

10Euclides van Alexandrie bewees dat als 2p − 1 een priemgetal is, dat dat 2p1(2p − 1) een perfect getal is. In 1747 bewees LeonhardEuler dat elk even perfect getal van die vorm is. Het is tot op heden niet bekend of er een perfect getal bestaat dat oneven is.

11 De eerste vijf paar paren bevriende getallen zijn (220, 284), (1184, 1210), (2620, 2924), (5020, 5564) en (6232, 6368). Leonhard Eulervond tussen 1747 en 1750 een zestigtal nieuwe bevriendegetallenparen. Anno 2007 waren er bijna twaalf miljoen paren bevriende getallenbekend. Het is niet bekend of er oneindig veel paren bevriende getallen zijn, en of er bevriende getallen bestaand die onderling ondeelbaarzijn.

XVI-26

Page 39: Deel XVI Getaltheorie (Hoofdstukken 1 en 2) (recto-verso)

U? Oefening 55 (gehele getallen met acht delers). Toon aan dat de gehele getallen met acht verschillende delersgegeven worden door

±p3 en ± pq waarbij p, q verschillende priemgetallen zijn.

Aanwijzing. Maak gebruik van Oefening 56(a).

U?? Oefening 56 (aantal positieve delers van een natuurlijk getal). Zij n ∈ N0 en stel dat de standaardontbindingvan n gegeven wordt door

n = pn11 pn2

2 . . . pnk

k

waarbij k ∈ N, ni ∈ N0 en p1 < p2 < . . . < pk positieve priemgetallen zijn.

(a) Toon aan dat het aantal positieve delers van n gegeven wordt door de formule

d(n) =

k∏i=1

(ni + 1).

(b) Bepaal het kleinste natuurlijke getal dat 15 positieve delers heeft.

Oefeningen bij §2.6

B Oefening 57. Zij a ∈ Z. Bewijs de volgende eigenschappen.

(a) ggd(a, 0) = |a| voor a 6= 0

(b) ggd(a, 1) = 1

(c) ggd(a, a) = |a| voor a 6= 0

(d) ggd(a, a+ 1) = 1

B Oefening 58. Bewijs dat elke twee verschillende positieve priemgetallen relatief priem zijn.

B Oefening 59 (Junior Wiskunde Olympiade 2013 eerste ronde).Wat is de grootste gemene deler van 2012 en 1220?

(A) 2 (B) 22 (C) 212 (D) 220 (E) 224

B? Oefening 60. Zij a ∈ Z en p een priemgetal. Bewijs dat

ggd(a, p) =

{p als p | a1 als p 6 | a.

B? Oefening 61. Zij n ∈ N. Bewijs dat ggd(35n+ 57, 45n+ 76) ∈ {1, 19}.

B? Oefening 62. Zij n ∈ N. Bewijs dat 5n+ 3 en 7n+ 4 onderling ondeelbaar zijn.

Oefening 63. Bepaal telkens de positieve grootste gemene deler van a en b met behulp van het algoritme van Euclides.Schrijf daarna die positieve grootste gemene deler als een lineaire combinatie van a en b.

B? (a) a = 1014 en b = 1404

B? (b) a = 2268 en b = 3444

B?? (c) a = 80 934 en b = 110 331

B?? (d) a = 147 231 en b = 839 160

Oefening 64. Zij a, b, c ∈ Z0. Bewijs de volgende eigenschappen.

B? (a) Als c | ab en ggd(a, c) = 1 dan is c | b.

B? (b) Als a | c en b | c en ggd(a, b) = 1 dan is ab | c.

B?? (c) ggd(ac, bc) = |c| ggd(a, b)

B?? (d) Als d = ggd(a, b) dan is ggd(a/d, b/d) = 1.

B?? Oefening 65. Zij a, b ∈ Z niet beide nul en d ∈ Z0. Bewijs dat de volgende uitspraken equivalent zijn:

(i) d is een grootste gemene deler van a en b,

(ii) del(d) = del(a) ∩ del(b),

V Oefening 66. Bewijs dat twee opeenvolgende kwadraten altijd relatief priem zijn.

V Oefening 67. Schrijf 20151977 als een lineaire combinatie van 71 en 83.

XVI-27

Page 40: Deel XVI Getaltheorie (Hoofdstukken 1 en 2) (recto-verso)

V Oefening 68 (Vlaamse Wiskunde Olympiade 2001 tweede ronde).De grootste gemene deler van 878 787 878 787 en 787 878 787 878 is

(A) 3 (B) 9 (C) 27 (D) 10 101 010 101 (E) 30 303 030 303

V? Oefening 69. Zij a, b ∈ Z0 en stel dat ggd(a, b) = 1. Vul aan en bewijs: ggd(a+ b, a− b) = . . .

V? Oefening 70. Zij n ∈ N0 en beschouw n+ 1 verschillende natuurlijke getallen, alle groter dan nul en kleiner of gelijkaan 2n. Toon aan dat er minstens een tweetal van deze getallen is die relatief priem zijn.

U Oefening 71 (grootste gemene deler van meer dan twee getallen). Zij a, b, c ∈ Z0. Een grootste gemene delervan a, b en c is een geheel getal d 6= 0 dat voldoet aan de volgende twee voorwaarden:

(1) d | a, d | b en d | c,(2) ∀e ∈ Z0 : e | a en e | b en e | c⇒ d | e.

Net zoals bij een grootste gemene deler van twee gehele getallen kan aangetoond worden dat d een grootste gemenedeler van a, b en c is als en slechts als dZ = aZ + bZ + cZ. De positieve grootste gemene deler van a, b en c wordt metggd(a, b, c) genoteerd. Analoog kan de grootste gemene deler van vier of meer gehele getallen worden gedefinieerd.

(a) Toon aan dat ggd(a, b, c)Z = ggd(a, b)Z + cZ.

(b) Bepaal ggd(13 320, 15 984, 19 980). Maak gebruik van je grafische rekenmachine.

U Oefening 72 (Diophantische vergelijkingen). Een Diophantische vergelijking is een vergelijking waarbij enkelde oplossingen worden onderzocht die gehele getallen zijn. De meest eenvoudige Diophantische vergelijkingen zijn delineaire veeltermvergelijkingen in een of meerdere onbekenden.Zij a, b, c ∈ Z0. Vul telkens de uitspraak aan en bewijs.

(a) De Diophantische vergelijking ax = c heeft oplossingen als en slechts als . . . | c.(b) De Diophantische vergelijking ax+ by = c heeft oplossingen als en slechts als . . . | c.

Oefeningen bij §2.7

B Oefening 73. Waar of vals? Beoordeel de volgende uitspraak. Indien vals, geef een tegenvoorbeeld. Indien waar,bewijs. Zij a, b ∈ Z0 en v ∈ Z0. Dan zijn de volgende uitspraken equivalent:

(i) v is een kleinste gemene veelvoud van a en b,

(ii) del(v) = del(a) ∪ del(b),

B? Oefening 74. Bepaal het positieve kleinste gemene veelvoud van 2204 en 4840 met behulp van het algoritme vanEuclides.

B? Oefening 75. Bepaal alle natuurlijke getallen waarvoor het positieve kleinste gemene veelvoud van dat getal en 30gelijk is aan 450.

B?? Oefening 76. Beschouw de gehele getallen a = 30 031 en b = 16 579.

(a) Bepaal ggd(a, b).

(b) Schrijf ggd(a, b) als een lineaire combinatie van a en b.

(c) Bepaal kgv(a, b).

V?? Oefening 77 (Nederlandse Wiskunde Olympiade 1998 finale).Van twee positieve gehele getallenm en n is het kleinste gemeenschappelijke veelvoud gelijk aan 133 866 = 2·3·3·3·37·67.Het verschil m− n is gelijk aan 189. Bereken m en n.

U Oefening 78 (kleinste gemene veelvoud van meer dan twee getallen). Zij a, b, c ∈ Z0. Een kleinste gemeneveelvoud van a, b en c is een geheel getal v 6= 0 dat voldoet aan de volgende twee voorwaarden:

(1) a | v, b | v en c | v,

(2) ∀d ∈ Z0 : a | d en b | d en c | d⇒ v | d.

Net zoals bij een kleinste gemene veelvoud van twee gehele getallen kan aangetoond worden dat v een kleinste gemeneveelvoud van a, b en c is als en slechts als vZ = aZ ∩ bZ ∩ cZ. Het positieve kleinste gemene veelvoud van a, b en cwordt met kgv(a, b, c) genoteerd. Analoog kan het kleinste gemene veelvoud van vier of meer gehele getallen wordengedefinieerd.

(a) Toon aan dat kgv(a, b, c)Z = kgv(a, b)Z ∩ cZ.

(b) Bepaal kgv(9009, 15 039, 18 942). Maak gebruik van je grafische rekenmachine.

XVI-28

Page 41: Deel XVI Getaltheorie (Hoofdstukken 1 en 2) (recto-verso)

U? Oefening 79 (eigenschap 2). Zij a, b ∈ Z0 en stel a = e pa11 p

a22 . . . pak

k en b = e′ pb11 pb22 . . . pbkk waarbij e, e′ ∈ Z∗,

k ∈ N, ai, bi ∈ N en p1 < p2 < . . . < pk positieve priemgetallen zijn. Bewijs dat

kgv(a, b) = pmax(a1,b1)1 p

max(a2,b2)2 . . . p

max(ak,bk)k .

Oefeningen bij §2.8

B Oefening 80. Zij a, n ∈ N, a ≥ 2 en stel dat p = an + 1 een priemgetal is. Bewijs dat a even is.

B Oefening 81 (Vlaamse Wiskunde Olympiade 2011 eerste ronde).Welk van de volgende tandwielen, die elk rond hun eigen (vaste) as draaien, maakt de meeste omwentelingen perminuut?

(A) I (B) II (C) III (D) IV (E) V

B Oefening 82. De omtrek van de voorwielen van een rijtuig is 2, 75 m, die van de achterwielen 3, 30 m. Welke isde kleinste afstand die het rijtuig moet afleggen als de voor- en achterwielen op hetzelfde ogenblik een geheel aantaltoeren moeten gedaan hebben?

B? Oefening 83. Gegeven is onderstaande ketting van tien tandwielen. Van elk tandwiel is het aantal tanden aangeduid.Bepaal de overbrengingsverhouding van het eerste tandwiel (links) naar het laatste tandwiel (rechts).

B? Oefening 84. Twee tandwielen werken in op elkaar. Het eerste tandwiel heeft een straal van 21 cm en maakt 635omwentelingen per minuut. De straal van het tweede tandwiel is gelijk aan 15 cm.

(a) Bepaal de hoeksnelheid (radialen per minuut) ω1 en ω2 van beide tandwielen.

(b) Na hoeveel minuten zullen beide tandwielen voor de eerste keer een geheel aantal omwentelingen gemaakt hebben?Geef de exacte waarde. Maak gebruik van je grafische rekenmachine.

B?? Oefening 85. De zijden van twee vierkante tuinen hebben elk een lengte die een geheel veelvoud is van 3 m, 3, 5 men 4 m. Het verschil tussen de twee oppervlakten12 bedraagt 3, 528 ha. Bereken de lengte van de zijde van elke tuin.

12Een hectare is een eenheid van oppervlakte van 100 m× 100 m = 10 000 m2 en wordt afgekort als ha. In een hectare gaan 100 aren.

XVI-29

Page 42: Deel XVI Getaltheorie (Hoofdstukken 1 en 2) (recto-verso)

V Oefening 86. Sinds 1997 is er in Groot-Brittanie een nieuw muntstuk van twee pondin omloop. De ene zijde beeldt Queen Elizabeth II uit, de ommezijde werd ontworpendoor Bruce Rushin en symboliseert de technologische vooruitgang door de eeuwenheen. Er is een gesloten ketting van 19 werkende tandwielen te zien (zie figuur).Welke blunder13 zag men blijkbaar over het hoofd?

V? Oefening 87. Zij a, n ∈ N, beide groter dan 1, en stel dat an − 1 een priemgetal is.Bewijs dat an − 1 een Mersennegetal is.

poster Vlaamse WiskundeOlympiade, 2006

V? Oefening 88. Vier tandwielen, met respectievelijk vijf, zeven, elf en dertien tanden,grijpen op elkaar in zoals in de figuur hiernaast. De vaste pijlen wijzen telkens naarhet getal op de bovenste tand, in de beginstand dus 2 0 0 6. Je kan aan elk van detandwielen draaien, om zo een ander getal in de bovenste positie te zetten. Uiteraardzullen de andere tandwielen dan meedraaien en ook een ander getal aanwijzen. Ishet mogelijk de huidige aanduiding 2 0 0 6 door draaiing te veranderen in 2 0 07? En zo ja: hoe vaak en in welke richting moet het bovenste tandwiel dan wordenrondgedraaid?

V?? Oefening 89. Als men de treden van een trap twee aan twee telt, blijft er 1 over. Teltmen ze drie aan drie, dan blijven er 2 over. Telt men ze vijf aan vijf, dan blijven er 4over. En telt men ze zeven aan zeven, dan blijven er 6 over. Bepaal alle mogelijkhedenvoor het aantal treden als je weet dat het aantal kleiner dan 1000 is.

U?? Oefening 90 (slijtage bij tandwielen). Is er bij samenwerkende tandwielen eentand oneffen of beschadigd, dan zal het andere tandwiel na verloop van tijd slijtagevertonen. Komt de beschadigde tand voortdurend met dezelfde tand of paar tandenin aanraking, dan zal die slijtage zich op een of enkele punten fixeren. Uiteraard wilmen dat de tandwielen zo lang mogelijk meegaan, dus moet men ervoor zorgen datde slijtage gelijkmatig optreedt. De optimale manier is om ervoor te zorgen dat erzoveel mogelijk tanden in aanraking met de beschadigde tand komen.

In onderstaande figuur werken twee tandwielen samen. Het kleinste tandwiel heeft z1 = 20 tanden en het grootstetandwiel heeft z2 = 30 tanden. De tanden zijn geletterd en genummerd van A tot en met T, resp. van 1 tot en met30. Tand A van het kleinste tandwiel drukt op een bepaald moment tegen tand 1 van het grote tandwiel.

(a) Met welke tanden komt tand A nog meer in aanraking als het een zeer groot aantal keer ronddraait?

(b) Zij r een geheel getal met 0 ≤ r < 30. Toon aan dat tand A in aanraking met tand r komt als en slechts alsr − 1 een veelvoud van 10 is.

(c) Stel dat het grote tandwiel z2 = 25, 29 of 21 tanden heeft. Met welke tanden komt tand A dan in aanraking?

(d) Toon aan dat elke tand van het ene tandwiel met elke tand van het andere tandwiel in aanraking komt alsen slechts als het aantal tanden van het ene tandwiel relatief priem is met het aantal tanden van het anderetandwiel.

13Een klachtenbrief aan de Royal Mint of Britain werd als volgt beantwoord: “We would, however, wish to emphasise that this is a coindesign symbolic of the development of technology and its success lies in visually representing a complex idea in an interesting and succinctfashion.” Naar schatting zijn momenteel zo’n 394 miljoen muntstukken van twee pond in omloop.

XVI-30

Page 43: Deel XVI Getaltheorie (Hoofdstukken 1 en 2) (recto-verso)

Antwoorden op geselecteerde oefeningen

Hoofdstuk 1

(1) (a) {3, 6}(b) {1, 2, 3, 4, 5, 6}(c) {1, 5}(d) {2, 4}(e) ∅(f) {1, 5}

(2) (a) vals

(b) waar

(c) waar

(d) vals

(e) waar

(f) vals

(g) waar

(h) vals

(3) (a) {−5, 5}(b) {1, 3, 5, 7, 9, . . .}(c) ∅

(4) (a) Q = ∅(b) P = ∅ en Q = ∅(c) P = Q

(d) P = Z en Q = ∅

(5) (D)

(6) (a) V = {−4,−3,−2,−1, 0, 1, 2, 3}(b) De verzameling V bevat een eindig aantal elementen en #V = 8.

(7) (a) vals

(b) vals

(c) vals

(d) vals

(e) waar

(f) vals

(g) vals

XVI-31

Page 44: Deel XVI Getaltheorie (Hoofdstukken 1 en 2) (recto-verso)

Hoofdstuk 2

(1) n ∈ {1,−1, 2,−2, 3,−3, 4,−4, 6,−6, 12,−12}

(2) (a) vals

(b) vals

(c) vals

(d) vals

(e) vals

(4) (C)

(5) (a) a = 10 of a = −10

(b) Zij a, b ∈ Z. Dan geldt: del(a) = del(b)⇔ a = b of a = −b.

(10) (a) Het quotient is 8 en de rest is 4.

(b) Het quotient is −9 en de rest is 3.

(c) Het quotient is −7 en de rest is 1.

(d) Het quotient is −3 en de rest is 3.

(e) Het quotient is −8 en de rest is 0.

(f) Het quotient is 0 en de rest is 15.

(11) {90, 91, 92, 93, 94}

(12) De deler is −25 en het deeltal is −600.

(13) 1098

(14) Het quotient is 12 en de rest is 8. Het gezochte geheel getal is 14.

(15) (a) −102 529

(b) −104 664

(17) De deler is gelijk aan 8.

(19) Het natuurlijk getal moet strikt tussen 40 en 99 liggen.

(20) (E)

(21) Ofwel is het deeltal gelijk aan 247 en de deler gelijk aan 19, ofwel is het deeltal gelijk aan 248 en de deler gelijkaan 18.

(23) (a) waar

(b) vals

(26) (C)

(30) De rest is gelijk aan 18.

(38) 4, 6, 8, 9, 10, 12, 14

(39) n ∈ {1,−1, p,−p}

(40) 2, 3, 5, 7, 11, 13, 17, 19, 23, 29, 31, 37, 41, 43, 47, 53, 59, 61, 67, 71, 73, 79, 83, 89, 97

(41)

(a = 40, b = 39

)of

(a = −40, b = −39

)of

(a = 40, b = −39

)of

(a = 40, b = −39

)(42) (a) n = 1

(b) n = 2

(c) n = 1 of n = 3

(d) Voor geen enkel natuurlijk getal n is p een priemgetal.

(43) (A)

(46) −2310

(47) n = 7350

XVI-32

Page 45: Deel XVI Getaltheorie (Hoofdstukken 1 en 2) (recto-verso)

(48) (a) −899 = (−1) · 29 · 31

(b) 1485 = 33 · 5 · 11

(c) −5820 = (−1) · 22 · 3 · 5 · 97

(d) 9075 = 3 · 52 · 112

(49) (D)

(50) 0

(56) (b) 144

(59) (E)

(63) (a) ggd(1014, 1404) = 78 = 7 · 1014− 5 · 1404

(b) ggd(2268, 3444) = 84 = 2 · 3444− 3 · 2268

(c) ggd(80 934, 110 331) = 123 = −223 · 110 331 + 304 · 80 934

(d) ggd 147 231, 839 160) = 189 = −263 · 103 005 + 184 · 147 231

(67) 20151977 = 71(−7 · 20151977

)+ 83

(6 · 20151977

)(68) (E)

(69) ggd(a+ b, a− b) = 1 of ggd(a+ b, a− b) = 2

(70) (b) 1332

(72) (a) a | c(b) ggd(a, b) | c

(74) 2 666 840

(75) 225 en 450

(76) (a) 59

(b) 59 = 53 · 30 031 + (−96) · 16 579

(c) 30 171 600

(77) m = 1998 en n = 1809

(78) (b) 1 234 431 198

(81) (C)

(82) 1650 m

(83) 3

(84) (a) ω1 = 1270π radialen/minuut en ω2 = 1778π radialen/minuut

(b) 1/127 minuten

(85) De zijde van de grootste tuin meet 252 m en de zijde van de kleinste tuin meet 168 m.

(88) Het bovenste tandwiel moet 1925 tanden in tegenwijzerzin gedraaid worden, oftewel 385 omwentelingen integenwijzerzin.

(89) {211, 421, 631, 841}

(90) (a) Tand A komt in aanraking met tanden 1, 21 en 11.

(c) Tand A komt respectievelijk in aanraking met tanden 1, 6, 11, 16, 21 en 1 en 1, 2, 3, 4, . . . , 21.

XVI-33

Page 46: Deel XVI Getaltheorie (Hoofdstukken 1 en 2) (recto-verso)
Page 47: Deel XVI Getaltheorie (Hoofdstukken 1 en 2) (recto-verso)

Referentielijst, bibliografie en websites

[1] M. Aigner, G.M. Ziegler, Proofs from the book, Springer, 1998.

[2] M. Alonso, E.J. Finn, Fundamentele natuurkunde deel 1 Mechanica, Delta Press, 1994.

[3] M. Alonso, E.J. Finn, Fundamentele natuurkunde deel 2 Elektromagnetisme, Delta Press, 1994.

[4] M. Alonso, E.J. Finn, Fundamentele natuurkunde deel 3 Golven, Delta Press, 1994.

[5] G.E. Andrews, The theory of partitions, Cambridge Mathematical Library, 1984.

[6] J. Anseeuw, J. De Langhe, P. Gevers, G. Roels, H. Vercauter, Nieuwe Delta 5 Analyse, Wolters, Leuven, 1994.

[7] J. Anseeuw, J. De Langhe, P. Gevers, G. Roels, H. Vercauter, Nieuwe Delta 6 Analyse Deel A, Wolters, Leuven,1994.

[8] J. Anseeuw, J. De Langhe, P. Gevers, G. Roels, H. Vercauter, Nieuwe Delta 5/6 Complexe getallen, Wolters,Leuven, 1994.

[9] J. Anseeuw, J. De Langhe, P. Gevers, G. Roels, H. Vercauter, Nieuwe Delta 5/6 Matrices en stelsels, Wolters,Leuven, 1994.

[10] J. Anseeuw, J. De Langhe, P. Gevers, G. Roels, H. Vercauter, Nieuwe Delta 5/6 Ruimtemeetkunde, Wolters,Leuven, 1994.

[11] D. Arnold, M. Butler, M. Haley, D. Harrow, A. Ives, S. Jackson, C. Kutil, T. Matsumoto, J.M. Prystowsky, T.Olsen, D. Tuttle, B. Wagner, Intermediate Algebra, College of the Redwoods Department of Mathematics, 2007.

[12] E. Aronson, T.D. Wilson, R.M. Akert, Social Psychology, Pearson Education, Limited, 2010.

[13] M. Artin, Algebra, Pearson Prentice Hall, 1991.

[14] F. Ayres , E. Mendelson, Schaum’s Outline of theory and problems of differential and integral calculus, McGraw-Hill, 1990.

[15] D. Batens, Logicaboek: praktijk en theorie van het redeneren, Antwerpen - Apeldoorn Garant, zevende druk, 2008.

[16] F. Beukers, Getaltheorie voor beginners, Epsilon Uitgaven 42, Utrecht, 2000.

[17] J. Billiet, H. Waege, Een samenleving onderzocht: Methoden van sociaal-wetenschappelijk onderzoek, UitgeverijDe Boeck nv, Antwerpen, 2005.

[18] P. Bogaert, F. Geeurickx, E. Willockx, R. Van Nieuwenhuyze, M. De Feyter, Van Basis tot Limiet 5 leerweg 6/8leerboek analyse 1: reele functies, Die Keure.

[19] D. Bollaerts, Wiskundige toelatingsexamens, Standaard Educatieve Uitgeverij, 1991.

[20] R. Bollens, H. Dooreman, L. Vermeiren, H. Van Bauwel, A. Florizoone-Dewachter, De wiskundepijl 6a bis, Uit-geverij Plantyn, Deune-Antwerpen, 1973.

[21] R. Bollens, H. Dooreman, L. Vermeiren, H. Van Bauwel, A. Florizoone-Dewachter, De wiskundepijl 6b, UitgeverijPlantyn, Deune-Antwerpen, 1973.

[22] R. Bollens, H. Dooreman, L. Vermeiren, H. Van Bauwel, A. Florizoone-Dewachter, De wiskundepijl 6d, UitgeverijPlantyn, Deune-Antwerpen, 1973.

[23] J. Bossaert, Curiosa Mathematica, 2014.

[24] P. E. Bourne, Ten Simple Rules for Making Good Oral Presentations PLoS Comput Biol 3(4): e77.doi:10.1371/journal.pcbi.0030077, 2007.

[25] A. Buijs, Statistiek om mee te werken, Wolters-Noodhoff, 2008.

xxii

Page 48: Deel XVI Getaltheorie (Hoofdstukken 1 en 2) (recto-verso)

[26] A. Buijs, K. De Bont, Statistiek om mee te werken opgaven en uitwerkingen, Wolters-Noodhoff, 2008.

[27] J. Buysse en M. Nachtegael, Wiskundig vademecum, Pelckmans, Kapellen, 2005.

[28] P. Carette, M. Guerry, P. Theuns, C. Vanderhoeft, Brugcursus wiskunde voor humane wetenschappen, VrijeUniversiteit Brussel, 1995.

[29] J. Casteels, D. De Vos, L. Goris, R. Rottiers, J. Salaets, F. Smessaert, L. Van den Broeck, Delta-T 5/6 Beknopteanalyse veeltermfuncties, Wolters, Leuven, 1994.

[30] J. Casteels, L. Goris, R. Rottiers, F. Smessaert, L. Van den Broeck, Delta-T 5/6 Complexe getallen, Wolters,Leuven, 1993.

[31] A. Clarysse en K. De Naeghel, Onderzoekscompetenties met Wiskunnend Wiske, Uitwiskeling 30/3, 4-15, 2014.

[32] P. Coppens, V. Descheemaeker, G. Gijbels, T. Jansen, P. Janssen, S. Janssens, P. Matthijs, F. Michiels, F.Roggeman, J. Schepers, Pienter leerboek wiskunde voor het derde jaar 5, Van In, 2006.

[33] P. Coppens, G. Finoulst, G. Gijbels, F. Roggeman, J. Schepers, R. Vanbuel, Pienter leerboek integraalrekeningen differentiaalvergelijkingen voor het zesde jaar 6/8, Van In, 2006.

[34] M. Crawford, The art of problem solving: Introduction to number theory, AoPS Incorporated, 2013.

[35] M. Crawford, The art of problem solving: Introduction to number theory solutions manual, AoPS Incorporated,2013.

[36] T. Crilly, 50 inzichten wiskunde Onmisbare basiskennis, Veen Magazines, Diemen, 2009.

[37] T. Crilly, De grote vragen Wiskunde, Veen Magazines, Diemen, 2011.

[38] P.W. Daly, H. Kopka, A guide to LaTeX Document preparation for beginners and advanced users, Addison-Wesley,1992.

[39] D. De Bock, J. Deprez, D. Janssens, G. Kesselaers, R. Op de Beeck, M. Roelens, J. Roels, Wiskunde vanuittoepassingen, Acco, Leuven, 1990.

[40] D. De Bock, H. Eggermont, M. Roelens, Context afgeleiden, Uitgeverij Plantyn, Deurne, 1989.

[41] D. De Bock, H. Eggermont, M. Roelens, Context beschrijvende statistiek, Uitgeverij Plantyn, Deurne, 1987.

[42] D. De Bock, H. Eggermont, M. Roelens, Context exponentiele en logaritmische functies, Uitgeverij Plantyn,Deurne, 1989.

[43] C. De Cock en N.J. Schons, Leerboek der rekenkunde voor het middelbaar onderwijs, De Procedure, Namen, 1962.

[44] C. De Cock en N.J. Schons, Logaritmentafels in vijf decimalen en bijtafels, De Procedure, Namen, 1962.

[45] M. de Gee, Wiskunde in werking, deel 1, Epsilon Uitgaven 48, 2002.

[46] M. de Gee, Wiskunde in werking, deel 2, Epsilon Uitgaven 49, 2002.

[47] M. de Gee, Wiskunde in werking, deel 3, Epsilon Uitgaven 50, 2002.

[48] M. de Gee, Wiskunde in werking, van A naar B, Epsilon Uitgaven 70, 2015.

[49] H.G. Dehling, J.N. Kalma, Kansrekening, Epsilon Uitgaven 36, Utrecht, 2005.

[50] G. Delaleeuw, Mathematiseren en oplossen van problemen voor de derde graad tso/kso, Cahiers T3 Europe Vlaan-deren nr.9, 2006.

[51] R. De Paepe, Basiswiskunde van het secundair onderwijs boek 2/A, Acco Leuven, 1989.

[52] R. De Paepe, Basiswiskunde van het secundair onderwijs boek 4/A, Acco Leuven, 1989.

[53] K. De Naeghel, Belastingverlaging en transformaties van functies, 8 oktober 2015 (aanvaard voor publicatie inUitwiskeling).

[54] K. De Naeghel, Benaderingen van het getal pi doorheen de geschiedenis van de wiskunde, 15 augustus 2013(aanvaard voor publicatie in Wiskunde & Onderwijs).

[55] K. De Naeghel, Enkele didactische wenken voor wiskundeonderwijs in de derde graad, print-on-demand onlinepublishing Lulu.com, 2009.

xxiii

Page 49: Deel XVI Getaltheorie (Hoofdstukken 1 en 2) (recto-verso)

[56] K. De Naeghel, Enkele didactische wenken voor wiskundeonderwijs in de derde graad, Wiskunde & Onderwijs150, 128-133, 2012.

[57] K. De Naeghel, Giscorrectie en optimaliseren van slaagkansen, Uitwiskeling 30/1, 2-7, 2014.

[58] K. De Naeghel, Het practicum wiskunde: coperatief aanleren van vaardigheden en attitudes, print-on-demandonline publishing Lulu.com, 2013.

[59] K. De Naeghel, Logaritmen en de zuurtegraad van een oplossing, 13 oktober 2015 (aanvaard voor publicatie inUitwiskeling).

[60] K. De Naeghel, MATHCOUNTS Trainer: een applicatie voor probleemoplossend denken in de klas, 8 november2015 (aanvaard voor publicatie in Uitwiskeling).

[61] K. De Naeghel, Over irrationale getallen en machten van pi, 3 april 2013 (aanvaard voor publicatie in Wiskunde& Onderwijs).

[62] K. De Naeghel, Steeds betere benadering voor het getal pi, Wiskunde & Onderwijs 149, 18-23, 2012.

[63] K. De Naeghel, Zinvol realiseren van competenties in de derde graad: visie en werkvormen, Wiskunde & Onderwijs161, 38-48 en 162, 113-123, 2015.

[64] K. De Naeghel, L. Gheysens, Pythagoras en lineaire transformaties, Wiskunde & Onderwijs 160, 312-313, 2014.

[65] K. De Naeghel, A. Timperman, Tekstopmaak met LaTeX Gebruik van de online editor Overleaf voor beginners,print-on-demand online publishing Lulu.com, 2014.

[66] K. De Naeghel en A. Timperman, Tekstopmaak met LaTeX Gebruik van de online editor Overleaf voor beginners,12 januari 2015 (aanvaard voor publicatie in Wiskunde & Onderwijs).

[67] K. De Naeghel en A. Timperman, Boekvoorstelling: Tekstopmaak met LaTeX Gebruik van de online editor Overleafvoor beginners, 7 november 2015 (aanvaard voor publicatie in Uitwiskeling).

[68] K. De Naeghel, L. Van den Broeck, SOHO Wiskunde Plantyn Lineaire Algebra I, Plantyn, 2014.

[69] K. De Naeghel, L. Van den Broeck, SOHO Wiskunde Plantyn Lineaire Algebra II, Plantyn, 2014.

[70] K. De Naeghel, L. Van den Broeck, P. Tytgat en B. Seghers, Boekvoorstelling: SOHO Wiskunde Plantyn Lineairealgebra I en II, 11 januari 2015 (aanvaard voor publicatie in Wiskunde & Onderwijs).

[71] J. De Langhe, N. Deloddere, L. De Wilde, N. De Wilde, P. Gevers, P. Tytgat, Delta 5/6 Analytische kansrekenen(6-8 lesuren), Wolters Plantyn, Mechelen, 2003.

[72] J. De Langhe, N. Deloddere, L. De Wilde, N. De Wilde, P. Gevers, K. Thaels, P. Tytgat, Delta 5/6 Ruimtemeet-kunde (6-8 lesuren), Wolters Plantyn, Mechelen, 2003.

[73] J. De Langhe, N. Deloddere, L. De Wilde, N. De Wilde, P. Gevers, P. Tytgat, Delta 5/6 Analyse deel 1 (6-8lesuren), Wolters Plantyn, Mechelen, 2003.

[74] J. De Langhe, N. Deloddere, L. De Wilde, N. De Wilde, P. Gevers, P. Tytgat, Delta 5/6 Analyse (4 lesuren),Wolters Plantyn, Mechelen, 2003.

[75] I. De Pauw, B. Masselis Wiskunde voor IT, Lannoo Campus, 2010.

[76] I. De Pauw, B. Masselis Wiskunde voor multimedia, Lannoo Campus, 2009.

[77] A. Depover, W. Herreman, N. Persoone, A. Vandekerckhove, Foton 4.3 - Elektriciteit, magnetisme, trillingen,Uitgeverij Pelckmans, 1988.

[78] A. Depover, W. Herreman, N. Persoone, A. Vandekerckhove, Fysica Vandaag 5.2/3, Uitgeverij Pelckmans, 1988.

[79] J. Deprez, H. Eggermont, P. Gevers, Nieuwe Delta 5/6 Analytische meetkunde deel A, Wolters, Leuven, 1993.

[80] J. Deprez, H. Eggermont, P. Gevers, Nieuwe Delta 5/6 Analytische meetkunde deel B, Wolters, Leuven, 1993.

[81] J. Deprez, H. Eggermont, E. Van Emelen, Met de krant in de hand, Uitwiskeling 23, Nr. 4, 14-49, 2007.

[82] J. Deprez, D. Janssens, Getaltheorie in het secundair onderwijs, Acco, Leuven, 1986.

[83] J. Deprez, G. Verbeeck, Onderzoekscompetenties wiskunde in de derde graad, DPB Brugge, 3 maart 2010.

[84] K. Devlin, Wiskunde Wetenschap van patronen en structuren, Natuur & Techniek, SEGMENT Uitgeverij, Beek,1998.

xxiv

Page 50: Deel XVI Getaltheorie (Hoofdstukken 1 en 2) (recto-verso)

[85] D. Domen, G. Finoulst, G. Gijbels, H. Put, J. Schepers, A. Vertenten, P. Weyenberg, Pienter leerboek reelefuncties precalculus voor het vijfde jaar 6/8, Van In, 2004.

[86] R. Donckels, L. Grootaert, D. Tant, L. Vandenbroucke, A. Van de Velde, V. Van de Walle, N. Vanhaverbeke, R.Vereecke, Richting: Reele functies - Analyse, Uitgeverij Pelckmans, 1988.

[87] R. Donckels, L. Grootaert, D. Tant, L. Vandenbroucke, A. Van de Velde, V. Van de Walle, N. Vanhaverbeke, R.Vereecke, Richting: Matrices - Stelsels, Uitgeverij Pelckmans, 1988.

[88] R. Donckels, L. Grootaert, D. Tant, L. Vandenbroucke, A. Van de Velde, V. Van de Walle, N. Vanhaverbeke, R.Vereecke, Richting: Reele vectorruimten - Ruimtemeetkunde, Uitgeverij Pelckmans, 1988.

[89] R. Donckels, L. Grootaert, D. Tant, L. Vandenbroucke, A. Van de Velde, N. Vanhaverbeke, V. Van de Walle, R.Vereecke, Richting: Analytische meetkunde, Uitgeverij Pelckmans, 1989.

[90] R. Donckels, L. Grootaert, D. Tant, L. Vandenbroucke, A. Van de Velde, N. Vanhaverbeke, V. Van de Walle, R.Vereecke, Richting: Complexe getallen, Uitgeverij Pelckmans, 1989.

[91] R. Donckels, L. Grootaert, D. Tant, L. Vandenbroucke, A. Van de Velde, N. Vanhaverbeke, V. Van de Walle, R.Vereecke, Richting: Goniometrie, Uitgeverij Pelckmans, 1989.

[92] R. Donckels, L. Grootaert, D. Tant, L. Vandenbroucke, A. Van de Velde, N. Vanhaverbeke, V. Van de Walle, R.Vereecke, Richting: Combinatieleer - Kansberekening, Uitgeverij Pelckmans, 1990.

[93] R. Donckels, L. Grootaert, D. Tant, L. Vandenbroucke, A. Van de Velde, N. Vanhaverbeke, V. Van de Walle, R.Vereecke, Richting: Analyse (minimum leerplan), Uitgeverij Pelckmans, 1990.

[94] R. Donckels, L. Grootaert, D. Tant, L. Vandenbroucke, A. Van de Velde, N. Vanhaverbeke, V. Van de Walle, R.Vereecke, Richting: Analyse (uitgebreid leerplan), Uitgeverij Pelckmans, 1990.

[95] R. Donckels, L. Grootaert, D. Tant, L. Vandenbroucke, A. Van de Velde, V. Van de Walle, N. Vanhaverbeke, R.Vereecke, Richting: Ruimtemeetkunde A, Uitgeverij Pelckmans, 1992.

[96] R. Donckels, L. Grootaert, D. Tant, L. Vandenbroucke, A. Van de Velde, V. Van de Walle, N. Vanhaverbeke, R.Vereecke, Richting: Telproblemen - Kansrekening - Statistiek, Uitgeverij Pelckmans, 1993.

[97] R. Donckels, L. Grootaert, D. Tant, L. Vandenbroucke, A. Van de Velde, V. Van de Walle, N. Vanhaverbeke, R.Vereecke, Richting: Analyse A, Uitgeverij Pelckmans, 1993.

[98] R. Donckels, L. Grootaert, D. Tant, L. Vandenbroucke, A. Van de Velde, V. Van de Walle, N. Vanhaverbeke, R.Vereecke, Richting: Beknopte analyse A, Uitgeverij Pelckmans, 1993.

[99] R. Donckels, L. Grootaert, D. Tant, L. Vandenbroucke, A. Van de Velde, V. Van de Walle, N. Vanhaverbeke, R.Vereecke, Richting: Matrices - Determinanten - Stelsels, Uitgeverij Pelckmans, 1993.

[100] R. Donckels, L. Grootaert, D. Tant, L. Vandenbroucke, A. Van de Velde, V. Van de Walle, N. Vanhaverbeke, R.Vereecke, Richting: Ruimtemeetkunde B, Uitgeverij Pelckmans, 1993.

[101] R. Donckels, L. Grootaert, D. Tant, L. Vandenbroucke, A. Van de Velde, V. Van de Walle, N. Vanhaverbeke, R.Vereecke, Richting: Analyse B, Uitgeverij Pelckmans, 1994.

[102] T. Dorissen, W. Jacquet, G. Sonck, Wiskundige basisvaardigheden, Uitgeverij VUBPRESS, 2008.

[103] W. Dunham, Euler: The master of us all, Dolciani Mathematical Expositions 22, 1999.

[104] W. Dunham, Journey through genius, Penguin books, 1990.

[105] W. Dunham, The calculus gallery, Princeton University Press, 2005.

[106] W. Dunham, The mathematical universe, John Wiley & Sons Inc., 1994.

[107] M. Du Sautoy, De getalmysteries, Uitgeverij Nieuwezijds, 2011.

[108] M. Du Sautoy, The music of the primes Searching to solve the greatest mystery in mathematics, HarperCollinsPublishers, 2003.

[109] B. Ernst, De interessantste bewijzen voor de stelling van Pythagoras, Epsilon Uitgaven 53, Utrecht, 2002.

[110] B. Ernst, Onmogelijke figuren, Librero, 2006.

[111] G. Finoulst, G. Gijbels, S. Janssens, H. Put, J. Schepers, A. Vertenten, P. Weyenberg, Pienter leerboek rijen enafgeleiden voor het vijfde jaar 6/8, Van In, 2005.

xxv

Page 51: Deel XVI Getaltheorie (Hoofdstukken 1 en 2) (recto-verso)

[112] F.R. Gantmacher, The theory of matrices volume one, American Mathematical Society, 1998.

[113] F.R. Gantmacher, The theory of matrices volume two, American Mathematical Society, 1998.

[114] M. Gardner, Sphere packing, Lewis Carroll and reversi, Cambridge University Press, 2009.

[115] M. Gardner, Sphere Packing, Lewis Carroll and Reversi, Cambridge University Press, 2009.

[116] G. Gijbels, E. Goemaere, D. Taecke, S. Wellecomme, Pienter leerwerkschrift voor de derde graad 2/3/4, Van In,2005.

[117] G. Gijbels, E. Govaert, M. Jaenen, S. Janssens, B. Sevenhant, I. Vanderstichel, P. Weyenberg, Pienter leerboekstatistiek I voor de derde graad 6/8, Van In, 2005.

[118] G. Gijbels, E. Govaert, M. Jaenen, S. Janssens, B. Sevenhant, I. Vanderstichel, P. Weyenberg, Pienter leerboektelproblemen en kansrekening statistiek II voor de derde graad 6/8, Van In, 2005.

[119] G. Gijbels, M. Jaenen, S. Janssens, B. Sevenhant, I. Vanderstichel, P. Weyenberg, Pienter leerboek ruimtemeet-kunde voor de derde graad 6/8, Van In, 2005.

[120] G. Gijbels, E. Govaert, M. Jaenen, S. Janssens, B. Sevenhant, I. Vanderstichel, K. Verzele, P. Weyenberg, Pienterleerboek complexe getallen en fractalen voor de derde graad 6/8, Van In, 2005.

[121] E. Goetghebeur, Statistiek, Universiteit Gent, uitgave 1997-1998.

[122] L.J. Goldstein, D.I. Schneider, M.J. Siegel, Finite mathematics and its applications, Prentice Hall, EnglewoodSliffs, New Jersey, 1988.

[123] W. Goldstein, W. Lewin, Gek op natuurkunde Van het begin van de regenboog tot het einde van de tijd: Een reislangs de wonderen van de wetenschap, Thomas Rap, Amsterdam, 2012.

[124] M. Goossens, F. Mittelbach, A. Samarin, The LATEX Companion, Addison-Wesley Publishing Compagny, 1994.

[125] R.L. Graham, D.E. Knuth, O. Patashnik, Concrete Mathematics, Addison-Wesley, 1994.

[126] E. Hairer, G. Wanner, Analysis by its history, Springer, 2000.

[127] P. Hammond, K. Sydsæter, Essential Mathematics for Economic Analysis, Prentice Hall, 2006.

[128] G.H. Hardy, Apologie van een wiskundige, Uitgeverij Nieuwezijds, 2011.

[129] J. Havil, Gamma, Princeton University Press, 2003.

[130] J. Havil, The irrationals, Princeton University Press, 2012.

[131] S. Hawking, God created the integers: The mathematical breakthroughs that changed history, Penguin Books,2005.

[132] N.J. Higham, Handbook of writing for the mathematical sciences, Society for Industrial and Applied Mathematics,Philadelphia, 1998.

[133] P. Hoffman, De man die van getallen hield Het verhaal van Paul Erds en de zoektocht naar de waarheid van dewiskunde, Bert Bakker, Amsterdam, 1999.

[134] L. Horsten, Einig, oneindig, meer dan oneindig, Epsilon Uitgaven 56, Utrecht, 2004.

[135] C. Impens, Analyse I, Universiteit Gent, uitgave 1996-1997.

[136] K. Ireland en M. Rosen, A classical introduction to modern number theory, Springer-Verlag, 1990.

[137] K. Janich, Linear Algebra, Springer-Verlag, 1994.

[138] S.L. Jones, Mathematical nuts, Norwood Press, 1932.

[139] D.W. Jordan, P. Smith, Mathematical techniques, Oxford University Press, 2002.

[140] D. Keppens, Algebra voor ingenieurs, Uitgeverij Acco, 2007.

[141] D. Keppens, Analyse voor ingenieurs, Uitgeverij Acco, 2006.

[142] M. Kindt, E. de Moor, Wiskunde in een notendop, Uitgeverij Bert Bakker, 2008.

[143] L. Kirkup, Experimental methods: An introduction to the analysis and presentation of data, Singapore, 1994.

[144] H. Kopla, P.W. Daly A guide to LATEX, Addison-Wesley Publishing Compagny, 1993.

xxvi

Page 52: Deel XVI Getaltheorie (Hoofdstukken 1 en 2) (recto-verso)

[145] E. Kreyszig, E.J. Norminton, Maple computer guide a self-contained introduction for advanced engineering ma-thematics, John Wiley & Sons, Inc., 2001.

[146] T. Kuijpers, C. Lybaert, SOHO Wiskunde Plantyn Groepentheorie, Plantyn, 2013.

[147] W. Ledermann en A.J. Weir, Introduction to group theory, Addison Wesley Longman, 1996.

[148] S. Lehoczky, R. Rusczyk, The art of problem solving: Volume 1: the Basics, AoPS Incorporated, 2008.

[149] S. Lehoczky, R. Rusczyk, The art of problem solving: Volume 1: the Basics solutions manual, AoPS Incorporated,2008.

[150] S. Lehoczky, R. Rusczyk, The art of problem solving: Volume 1: and beyond, AoPS Incorporated, 2008.

[151] S. Lehoczky, R. Rusczyk, The art of problem solving: Volume 1: and beyond solutions manual, AoPS Incorpo-rated, 2008.

[152] J. Levy, Scheikunde van zuren en basen tot chemische polariteit, Librero, 2012.

[153] S. Lipschutz, Schaum’s Outline of linear algbebra, McGraw-Hill, 1991.

[154] J. Lyczak, Q. Puite, B. van Dalen, Finaletraining Nederlandse wiskunde olympiade met uitwerkingen, ISBN978-90-357-1800-5, 2011.

[155] M. Mashaal, Bourbaki, Veen Magazine, Amsterdam, 2009.

[156] E. Mathijs, Schrijfstijl wetenschappelijke tekst, KU Leuven, 2006.

[157] J.T McClave, P.G. Benson, T. Sincich, S. Knypstra, Statistiek: een inleiding, elfde editie, Pearson EducationBenelux, 2011.

[158] R. Mersch, Oogklepdenken, De Bezige Bij Antwerpen, 2012.

[159] B. Michels, Getaltheorie een introductie, 2015.

[160] L. Motmans, Inhaalcursus wiskunde voor het eerste kandidaatsjaar Toegepaste Economische Wetenschappen enHandelsingenieur, Limburgs Universitair Centrum, 2002.

[161] M. Nachtegael, Data-Analyse I: Wiskundige Principes, Faculteit Geneeskunde en Gezondheidswetenschappen,Universiteit Gent, 2009.

[162] E. Nauwelaerts, Basiswiskunde voor informatica 2, Universiteit Hasselt, 2002.

[163] E. Nauwelaerts, Redeneren en structureren, Universiteit Hasselt, 2005.

[164] I. Newton, Method of fluxions, 1736.

[165] B.M. Oliver, Heron’s remarkable triangular area formula, Mathematics Teacher 86, 161-163, 1993.

[166] J.M.H. Olmsted, C.G. Townsend, On the Sum of Two Periodic Functions, The Two-Year College MathematicsJournal, Vol. 3, No. 1, 33-38, 1972.

[167] J.P. Ottoy, Wiskunde algebra analystische meetkunde, Universiteit Gent,

[168] D. Patrick, The art of problem solving: Calculus, AoPS Incorporated, 2010.

[169] D. Patrick, The art of problem solving: Calculus solutions manual, AoPS Incorporated, 2010.

[170] D. Patrick, The art of problem solving: Introduction to counting & probability, AoPS Incorporated, 2007.

[171] D. Patrick, The art of problem solving: Introduction to counting & probability manual, AoPS Incorporated, 2007.

[172] L. Papula, Wiskunde voor het hoger technisch onderwijs deel 1, Academic Service, 2009.

[173] L. Papula, Wiskunde voor het hoger technisch onderwijs deel 2, Academic Service, 2009.

[174] J.A. Paulos Ongecijferdheid, Uitgeverij Ooievaar Amsterdam, 1999.

[175] R. Penrose, The emperor’s new mind Concerning computers, minds and the laws of physics, Oxford UniversityPress, 1989.

[176] R. Penrose, The road to reality A complete guide to the laws of the universe, Vintage Books, London, 2005.

[177] H. Pfaltzgraff, Spijker 1 Rekenen, Epsilon Uitgaven, Utrecht, 2009.

xxvii

Page 53: Deel XVI Getaltheorie (Hoofdstukken 1 en 2) (recto-verso)

[178] H. Pfaltzgraff, Spijker 2 Algebra, Epsilon Uitgaven, Utrecht, 2009.

[179] H. Pfaltzgraff, Spijker 3 Functies, Epsilon Uitgaven, Utrecht, 2009.

[180] H. Pfaltzgraff, Spijker 4 Differentieren, Epsilon Uitgaven, Utrecht, 2009.

[181] H. Pfaltzgraff, Spijker 5 Integreren, Epsilon Uitgaven, Utrecht, 2009.

[182] H. Pfaltzgraff, Spijker 6 Statistiek, Epsilon Uitgaven, Utrecht, 2009.

[183] H. Pfaltzgraff, Spijker 7 Goniometrie en vectoren, Epsilon Uitgaven, Utrecht, 2009.

[184] C.A. Pickover, Het wiskunde boek, Librero Nederland, 2010.

[185] G. Polya, How to solve it, Princeton University Press, 1945.

[186] H. Reuling, J. Reuling, Tandwielen en overbrengingen wiskunde D havo-5, 2010.

[187] S.E. Rigdon, E.J. Purcell, D. Varberg, Calculus, Pearson Prentice Hall, 2007.

[188] J. Rosenhause, The Monty Hall problem, Oxford University Press, 2009.

[189] R. Rusczyk, The art of problem solving: Precalculus, AoPS Incorporated, 2009.

[190] R. Rusczyk, The art of problem solving: Precalculus solutions manual, AoPS Incorporated, 2009.

[191] R. Rusczyk, M. Crawford, The art of problem solving: Intermediate Algebra, AoPS Incorporated, 2008.

[192] R. Rusczyk, M. Crawford, The art of problem solving: Intermediate Algebra solutions manual, AoPS Incorpora-ted, 2008.

[193] N.J. Schons, Exercices d’arithmologie, La Procedure, Namur, 1938.

[194] S. Singh, De oerknal De belangrijkste wetenschappelijke ontdekking ooit, De Arbeiderspers, Amsterdam, 2005.

[195] M.R. Spiegel, Schaum’s Outline of theory and problems of advanced calculus, McGraw-Hill, 1962.

[196] E. Steiner, The Chemestry Maths Book, Oxford University Press, 2008.

[197] I. Steward, Concepts of modern mathematics, Dover Publication, 1975.

[198] D.J. Struik, Geschiedenis van de wiskunde, Het Spectrum, 1990.

[199] K. Stulens, Herhalingslessen wiskunde Voorbereiding tot het eerste kandidaatsjaar wiskunde-natuurkunde-informatica-kennistechnologie, Limburgs Universitair Centrum, 2000.

[200] J. Tinbergen, Vertragingsgolven en levensduurgolven, Strijdenskracht door Wetensmacht pp. 143 - 150 (1938).

[201] J.P. Van Bendegem, Inleiding tot de moderne logica en wetenschapsfilosofie: een terreinverkenning, VUBPRESS,Brussel, 2001.

[202] J. van de Craats, Vectoren en Matrices, Epsilon Uitgaven 45, Utrecht, 2005.

[203] J. van de Craats, R. Bosch Basisboek wiskunde, Pearson Education, 2010.

[204] M. Van den Berghe, Inleiding tot zelfstandig onderzoek, Onze-Lieve-Vrouwecollege Assebroek, 2006.

[205] M. Van den Berghe, OZo! Onderzoeken doe je zo, Plantyn, Mechelen, 2014.

[206] V. van der Noort, Getallen zijn je beste vrienden, Athenaeum - Polak & Van Gennep, Amsterdam, 2011.

[207] B.L. Van der Waerden, Algebra Volume 1, Springer-Verlag, 1991.

[208] B.L. Van der Waerden, Algebra Volume 2, Springer-Verlag, 1991.

[209] J. Van Geel, Commutatieve ringtheorie, Universiteit Gent, 1997.

[210] Th.M. van Pelt, R.B.J. Pijlgroms, W.V. Smeets, J.L. Walter, Wiskunde voor het hoger onderwijs deel 1, Wolters-Noordhoff, 2006.

[211] Th.M. van Pelt, R.B.J. Pijlgroms, W.V. Smeets, J.L. Walter, Wiskunde voor het hoger onderwijs deel 1 uitwer-kingen en extra, praktijkgerichte vraagstukken, Wolters-Noordhoff, 2009.

[212] P. Wauters, Wiskunde Deel 1, Faculteit Toegepaste Economische Wetenschappen, Universiteit Hasselt, 2002.

xxviii

Page 54: Deel XVI Getaltheorie (Hoofdstukken 1 en 2) (recto-verso)

[213] D.T. Whiteside, The Mathematical Papers of Isaac Newton, Vol. 1, 1664-1666, Ed. Cambridge University Press,New York, 1967.

[214] A.J. Wiles, Modular elliptic curves and Fermat’s Last Theorem, Annals of Mathematics, 141, 443-551, 1995.

[215] H. Wussing, Geschiedenis van de wiskunde Vanaf de wetenschappelijke revolutie tot de twintigste eeuw, VeenMagazines, Diemen, 2010.

[216] Website ADSEI, http://statbel.fgov.be/ .

[217] Website American Mathematical Association of Two-Year Colleges - Students Mathematics League,http://www.amatyc.org/SML/ .

[218] Website American Mathematics Competitions, http://amc.maa.org/ .

[219] Website D. Arnold, M. Butler, M. Haley, D. Harrow, A. Ives, S. Jackson, C. Kutil, T. Matsumoto, J.M. Prysto-wsky, T. Olsen, D. Tuttle, B. Wagner, Intermediate Algebra http://facweb.northseattle.edu/dli/IntAlgebraText/

.

[220] Website arXiv, http://xxx.lanl.gov/ .

[221] Website J. Bossaert, http://users.ugent.be/∼jebossae/ .

[222] Website carrieretijger, http://www.carrieretijger.nl/ .

[223] Website C. Cambre, http://users.telenet.be/chris.cambre/chris.cambre/ .

[224] Website J. Claeys, http://home.scarlet.be/math/ .

[225] Website M. Davidson, J. Dethridge, H. Kociemba, T. Rokicki, God’s Number is 20, http://www.cube20.org/ .

[226] Website K. De Naeghel, http://www.koendenaeghel.be/ .

[227] Website GeoGebra, http://www.geogebra.org/ .

[228] Website GeoGebraTube, http://www.geogebratube.org/ .

[229] Website kennislink.nl, http://www.kennislink.nl/publicaties/wiskundige-bijsluiter-van-opiniepeilingen .

[230] Website Leerplan A derde graad ASO: studierichtingen met component wiskunde D/2004/0279/019,http://ond.vvkso-ict.com/leerplannen/doc/Wiskunde-2004-019.pdf .

[231] Website leren.nl, http://www.leren.nl/cursus/leren−en−studeren/portfolio/wat-is-portfolio.html/ .

[232] Website leren.nl, http://www.leren.nl/cursus/professionele-vaardigheden/presentatie/ .

[233] Website Nederlandse Wiskunde Olympiade, http://www.wiskundeolympiade.nl/ .

[234] Website niutec.nl Tandwielen, http://www.niutec.nl/Mechanica/HTML5/tandwielOverbrenging.htm/ .

[235] Website McGraw-Hill Professional, http://www.mhprofessional.com/templates/index.php?cat=145/ .

[236] Website ticalc.org voor het downloaden van programma’s op de grafische rekenmachine,http://www.ticalc.org/pub/83plus/basic/math/ .

[237] Website USolv-IT, http://www.usolvit.be/ .

[238] Website Vlaamse Wiskunde Olympiade, http://www.vwo.be/ .

[239] Website Wikipedia, http://en.wikipedia.org/ en http://en.wikipedia.org/ .

[240] Website wiskunde B-dag, http://www.fisme.science.uu.nl/wisbdag/ .

xxix

Page 55: Deel XVI Getaltheorie (Hoofdstukken 1 en 2) (recto-verso)
Page 56: Deel XVI Getaltheorie (Hoofdstukken 1 en 2) (recto-verso)

c©2013 Koen De Naeghelroyalty percentage: 0%

Wiskunde In zicht is een cursus wiskunde bestemd voor leerlingen vande derde graad algemeen secundair onderwijs in de studierichtingen metzes of acht wekelijkse lestijden wiskunde. Het werd ontworpen vanuit debehoefte aan een natuurlijke en correcte, maar toch haalbare benaderingvan basisconcepten in de wiskunde. Er werd bewust gekozen voor

3 een invulcursus, zodat de leerling ervaart hoe bepaalde oplossingsme-thoden opgebouwd worden, terwijl de leerkracht nog voldoende vrijheidheeft die methoden op zijn of haar eigen manier aan te brengen;

3 differentiatie door de oefeningen in verschillende niveaus op te delen,zodat de leerling zelfstandig kan werken, afgestemd op eigen niveau enwerktempo (Deel Portfolio wiskunde);

3 ontwikkelen van vaardigheden en attitudes waarbij getracht werdde kwaliteit van de wiskunde te respecteren (Deel Problem Solvingwiskunde, Deel Practicum wiskunde, Deel GeoGebra en Deel Maple).